microbiology ii

88
An 8-year-old boy is taken to a pediatrician because of behavioral changes, mild intellectual deterioration, and "laziness." Over the next several months the boy develops increasing clumsiness and periodic, involuntary, jerky movements every 3 to 6 seconds. Visual deterioration is apparent upon visual field testing, and optic atrophy is evident on funduscopic examination. Cerebrospinal fluid studies do not show significant pleocytosis, but oligoclonal bands of IgG are present on CSF electrophoresis. The electroencephalogram shows periodic discharges that are synchronous with the periods of myoclonus. Computed tomography (CT) of the head shows low-density white matter lesions and cerebral atrophy. At the age of 10, the boy dies. Prior infection with which of the following agents was probably related to the patient's condition? A. Measles virus B. Mumps virus C. Papilloma virus D. Poliovirus E. Varicella Explanation: The correct answer is A. The rare disease illustrated is subacute sclerosing panencephalitis (SSPE), which typically presents as in the question stem. SSPE appears to be due to a combination of persistent, possibly abnormal measles virus and to autoimmune damage caused by antibodies directed against the virus. Many patients developing SSPE have had measles at 2 years of age or younger; there is typically a six-year interval between measles infection and symptom development. A small proportion of cases have followed vaccination with live measles virus. Unfortunately, no effective therapy has been developed, although some experimental work with drugs such as isoprinosine shows some promise. Post-infectious encephalomyelitis, rather than SSPE, can follow mumps (choice B) or varicella (choice E). Latent

Upload: jaime-sarmiento-zegarra

Post on 04-Apr-2015

683 views

Category:

Documents


3 download

TRANSCRIPT

Page 1: Microbiology II

An 8-year-old boy is taken to a pediatrician because of behavioral changes, mild intellectual deterioration, and "laziness." Over the next several months the boy develops increasing clumsiness and periodic, involuntary, jerky movements every 3 to 6 seconds. Visual deterioration is apparent upon visual field testing, and optic atrophy is evident on funduscopic examination. Cerebrospinal fluid studies do not show significant pleocytosis, but oligoclonal bands of IgG are present on CSF electrophoresis. The electroencephalogram shows periodic discharges that are synchronous with the periods of myoclonus. Computed tomography (CT) of the head shows low-density white matter lesions and cerebral atrophy. At the age of 10, the boy dies. Prior infection with which of the following agents was probably related to the patient's condition?

A. Measles virus

B. Mumps virus

C. Papilloma virus

D. Poliovirus

E. Varicella

Explanation:

The correct answer is A. The rare disease illustrated is subacute sclerosing panencephalitis (SSPE), which typically presents as in the question stem. SSPE appears to be due to a combination of persistent, possibly abnormal measles virus and to autoimmune damage caused by antibodies directed against the virus. Many patients developing SSPE have had measles at 2 years of age or younger; there is typically a six-year interval between measles infection and symptom development. A small proportion of cases have followed vaccination with live measles virus. Unfortunately, no effective therapy has been developed, although some experimental work with drugs such as isoprinosine shows some promise.

Post-infectious encephalomyelitis, rather than SSPE, can follow mumps (choice B) or varicella (choice E). Latent infection with the varicella virus (choice E) causes shingles (herpes zoster).

Papilloma viruses (choice C) are associated with warts. They do not usually infect the brain.

Poliovirus (choice D) causes gastrointestinal disturbances, viremia, and paralysis.

A 57-year-old man presents with an episode of shaking chills the previous night. He has now developed right-sided pleuritic chest pain, fever, sweats, malaise, purulent sputum, and mild hemoptysis. On examination,

Page 2: Microbiology II

the patient is diaphoretic but alert, with right basilar rales. Chest X-ray films show a right lower lobe infiltrate with blunting of the right costophrenic angle. Why is this patient's sputum filled with pus?

A. Teichoic acids and peptidoglycan are chemotactic for neutrophils

B. The capsule of the causative agent is chemotactic for neutrophils

C. The causative agent is an intracellular organism

D. The causative agent is beta hemolytic

E. The organism produces an IgA protease

Explanation:

The correct answer is A. The answer to this question requires that the student realize that pus consists of bacteria and dead and dying neutrophils. This fact, taken along with the highly characteristic case history, reflects that the patient has a typical pneumonia. In the United States, the most common agent of this would be Streptococcus pneumoniae, a gram-positive extracellular pathogen rich in teichoic acids and peptidoglycan, which elicit the neutrophilic exudate.

The capsule of this organism is a polysaccharide and primarily elicits an antibody response rather than attracting neutrophils (choice B).

Streptococcus pneumoniae is an extracellular, not an intracellular (choice C) organism.

Streptococcus pneumoniae is alpha hemolytic, not beta hemolytic (choice D).

Streptococcus pneumoniae does produce an IgA protease (choice E) that enhances the ability of the organism to infect the respiratory mucosa, but this does not contribute to pus formation.

A 48-year-old waitress presents to a physician with malaise, loss of appetite, nausea, moderate fever, and jaundice. Laboratory tests indicate a marked increase in serum transaminases. Serology for hepatitis viruses is performed and indicates positive results for the presence of HBsAg, HBc IgM antibody, and HCV antibody. Antibody tests for HBsAb and HAV are negative. The results indicate:

A. A dual infection of HBV and HAV

B. Chronic hepatitis A infection

C. Chronic hepatitis B infection

Page 3: Microbiology II

D. Hepatitis C infection

E. The presence of an acute HBV infection.

Explanation:

The correct answer is E. The presence of hepatitis B surface antigen (HBsAg) along with hepatitis B core IgM antibody (HBc IgM Ab), and the absence of hepatitis B surface antibody (HBsAb) indicates the presence of the early stages of an acute infection with Hepatitis B. The presence of antibody to Hepatitis C (HCV) only indicates exposure, but not a specific time of exposure; however, 85% of patients who are infected with HCV develop chronic infections, indicating that this patient has an 85% chance of having a dual infection with HBV and HCV. The acute or chronic HCV infection can be confirmed by PCR.

A dual infection of HBV and HAV (choice A) is not plausible since the IgM anti-HAV serology is negative.

Hepatitis A does not cause chronic disease (choice B).

Chronic HBV infection (choice C) is unlikely because the patient has HBc IgM Ab, which is characteristic of an acute infection, rather than a chronic infection.

Hepatitis C infection (choice D) is not confirmed by these data because the presence of HCV Ab only indicates exposure to the virus, and not the state of infection. This could be caused by exposure at some earlier time (the elevated serum transaminases might be due to HBV infection). An active or chronic HCV infection can only be confirmed by PCR.

An 8-year-old girl is brought to the pediatrician for a severe sore throat. The pediatrician prescribes penicillin and sends the girl home. Later that day, she develops a diffuse maculopapular rash, shortness of breath, and wheezing. Her parents take her to the emergency department, where she is diagnosed with anaphylaxis and treated successfully with epinephrine. Three months later, the girl has dysuria and urinary frequency. Urine Gram's stain reveals numerous gram-negative rods. Which of the following antibiotics could be safely given to this patient?

A. Ampicillin

B. Aztreonam

C. Cefoperazone

Page 4: Microbiology II

D. Cephalexin

E. Methicillin

F. Oxacillin

G. Ticarcillin

Explanation:

The correct answer is B. Aztreonam is a monobactam. Since the basic ring structure is different from penicillins, there is no cross-allergenicity, and it can be safely given to those who have had severe reactions to penicillins. Aztreonam is highly active against gram-negative bacteria, but has no activity against gram-positive bacteria or obligate anaerobes. It is not β-lactamase resistant.

In general, there is cross-allergenicity between all the penicillins, since most people react to breakdown products of the β-lactam ring structure common to all penicillins. Patients who have experienced an allergic reaction to a penicillin may also be sensitive to cephalosporins.

Ampicillin (choice A) is a third-generation penicillin. It is a broad-spectrum penicillin that is active against certain gram-negative and gram-positive bacteria. It is also one of the main antibiotics (along with clindamycin) that can lead to antibiotic-induced pseudomembranous colitis.

Cefoperazone (choice C) is a third-generation cephalosporin. It is a broad-spectrum cephalosporin that is active against many gram-negative and gram-positive bacteria. It contains a methylthiotetrazole side chain that can cause a vitamin K deficiency and disulfiram-like reaction to alcohol. Many people also experience diarrhea as a side effect. It is a drug of choice in people with impaired renal function because 60% is eliminated by the biliary route.

Cephalexin (choice D) is a first-generation cephalosporin. It is mainly active against gram-positive bacteria.

Methicillin (choice E) is a second-generation penicillin. It is active against many gram-positive organisms. When given in high doses for more than 2 weeks, it can cause interstitial nephritis, hepatitis, and neutropenia.

Oxacillin (choice F) is a second-generation penicillin. It is used against many gram-positive organisms. When used in high dose for more than 2 weeks, it can cause hepatitis and neutropenia.

Ticarcillin (choice G) is a fourth-generation penicillin. Fourth-generation penicillins are extended-spectrum

Page 5: Microbiology II

agents that are active against many gram-positive and gram-negative bacteria, including Pseudomonas and many Enterobacteriaceae. Ticarcillin is given with clavulanic acid, a β-lactamase inhibitor. Since ticarcillin is a disodium salt, it causes a large salt load, which can lead to salt retention and hypokalemia.

A 38-year-old woman with a history of multiple sexual partners is most at risk for which of the following?

A. Bladder carcinoma

B. Cervical carcinoma

C. Endometrial carcinoma

D. Ovarian carcinoma

E. Rectal carcinoma

Explanation:

The correct answer is B. In a sense, cervical carcinoma can be considered a "sexually transmitted disease," since human papilloma virus (typically strains HPV-16 or HPV- 18) is transmitted venereally. In our culture, HPV-related dysplasia of the cervix is common, even in the 18-28 year old group, and carcinomas of the cervix (more rare now than in the past due to screening and aggressive therapy of dysplasia) can occur in the 35-45 age group. The other cancers listed typically occur in older age groups and are not as clearly related to venereal agents.

A Brazilian immigrant is hired at a meat-packing plant, and undergoes an employment physical. Chest x-ray demonstrates a patchy, bilateral pneumonia and a lung mass, and he is referred to a specialist. Biopsy of the mass demonstrates fungal organisms with a few very distinctive "pilot's wheel" yeast forms. Which of the following is the most likely diagnosis?

A. Blastomycosis

B. Coccidioidomycosis

C. Histoplasmosis

D. Paracoccidioidomycosis

E. Sporotrichosis

Page 6: Microbiology II

Explanation:

The correct answer is D. Latin America and "pilot's wheel" budding yeast are clues for paracoccidioidomycosis. This disease is caused by Paracoccidioides brasiliensis, a dimorphic fungus that is found as a multiply-budding yeast in tissues.

Clues for questions about blastomycosis (choice A) would include spending time in states east of the Mississippi River and fairly large yeast.

Clues for questions about coccidioidomycosis (choice B) would be spending time in the southwestern deserts of the United States and spherules filled with endospores.

Clues for questions about histoplasmosis (choice C) would be spending time in the Ohio, Mississippi, and Missouri River Valleys and finding tiny yeast forms in macrophages.

Clues for questions about sporotrichosis (choice E) would be skin lesions in rose gardeners.

A psychotic, indigent man with a history of multisubstance abuse has been involuntarily hospitalized for 1 week. Because of persistent diarrhea, stools are sent for ova and parasites, revealing numerous granular, spherical, thin-walled cysts measuring 10-20 µm in diameter. Trichrome stains show up to four nuclei in most of the cysts. These finding are consistent with an infection by which of the following organisms?

A. Cryptosporidium parvum

B. Dientamoeba fragilis

C. Entamoeba histolytica

D. Giardia lamblia

E. Isospora belli

Explanation:

The correct answer is C. Entamoeba are relatively common enteric pathogens that can produce asymptomatic infection or more severe disease characterized by mucosal ulcerations and submucosal spread causing abdominal distress and liquid stools. Stools may show either trophozoite forms or the typical spherical cysts. Several species of Entamoeba are seen, including Entamoeba coli and E. hartmanni. E. histolytica cysts characteristically are spherical in shape, 10-20 µm in diameter, and have granular cytoplasm containing 1, 2, or 4 nuclei.

Page 7: Microbiology II

Cryptosporidium parvum(choice A) infections occur in the immunocompromised population and may cause severe diarrhea. The organism presents as minute (2-5 µm) intracellular spheres or arc-shaped merozoites under normal mucosa, and can be difficult to appreciate by light microscopy. Cysts in the stool are too small (4-5 µm) to be confused with Entamoeba.

Dientamoeba fragilis(choice B) is an intestinal amoeba that also may produce an infectious diarrhea. It does not have a cyst form, and only the trophozoite forms are seen in stools.

Giardia lamblia(choice D) is a flagellate protozoan that infects the stomach and small intestine when contaminated water is ingested. Diagnosis is usually made by examining duodenal contents; however, the stools may contain the oval or elliptical cysts, which are thick-walled and measure 8-14 µm in diameter. Spherical cysts are not seen in Giardia infection.

Isospora belli(choice E) produces self-limited intestinal infections mostly in the tropics, where fever and diarrhea may last weeks to months. The stool-borne cysts are much larger than Entamoeba (30 x 15 µm), are asymetrical, and are typically almond-shaped.

An autopsy is performed on a man who suddenly began vomiting voluminous quantities of blood and exsanguinated.

Page 8: Microbiology II

The man's stomach is presented in the accompanying photograph. Which of the following organisms is most likely implicated in the pathogenesis of this disease?

A. Cryptosporidium parvum

B. Entamoeba histolytica

C. Escherichia coli

D. Helicobacter pylori

E. Mycobacterium tuberculosis

Explanation:

The correct answer is D. The gross photograph shows a stomach with a perforated peptic ulcer. Perforation of a peptic ulcer is potentially fatal, because of either peritonitis with sepsis or sudden exsanguination (if the perforation damages one of the many arteries of the stomach). Peptic ulcer disease, gastritis, and possibly gastric carcinoma and gastric lymphoma have been strongly associated with Helicobacter pylori colonization of the mucus layer covering the gastric mucosa. Colonization is associated with destruction of the mucus layer, thereby destroying its protective function.

Cryptosporidium parvum(choice A) causes diarrhea that is severe in immunocompromised patients.

Entamoeba histolytica(choice B) produces dysentery-like symptoms or can cause liver abscess.

Escherichia coli(choice C) causes a variety of diarrheal diseases and can infect the bladder and soft tissues.

Mycobacterium tuberculosis(choice E) causes tuberculosis, characterized by granuloma formation, especially in the lungs.

A 34-year-old HIV-positive man without previous opportunistic infections presents complaining of dyspnea with daily activity. He states that he has had a mild cough and fever but denies having had chills, sputum production, or chest discomfort. Physical examination is remarkable for oral thrush and a few small, nontender cervical lymph nodes. A chest x-ray film reveals bilateral interstitial infiltrates, and bronchoalveolar lavage reveals small silver-staining cysts. In which other patient population in the U.S. is this organism a frequent cause of a life-threatening pneumonia?

Page 9: Microbiology II

A. Bone marrow transplant patients

B. Hospitalized adults on antibiotic therapy

C. Late-term pregnant women

D. Normal adults in periods of stress

E. Premature infants

F. Preschool children

G. Sexually active adolescents

H. Third-trimester fetuses

Explanation:

The correct answer is E. This patient has Pneumocystis carinii pneumonia, the most common atypical pneumonia in AIDS patients. The next most common pool of at-risk individuals consists of premature infants.

Bone marrow transplant patients (choice A) would be immunologically compromised, and might be a second at-risk group (behind premature infants), but would not be the first choice on this list.

Hospitalized adults on antibiotic therapy (choice B) might be slightly immunologically compromised or stressed, but are not the major patient pool at risk for this infection in the U.S.

Late-term pregnant women (choice C) are not immunologically compromised and would not be at risk.

Normal adults in periods of stress (choice D) are not particularly susceptible to opportunistic pathogens.

Preschool children (choice F) have normal rates of exposure to P. carinii, as would any adult, but are not susceptible to serious pneumonia with this agent unless they are immunologically suppressed.

Sexually active adolescents (choice G) would have the same infection rates as normal infants and adults, but again would not be susceptible to life-threatening pneumonia. P. carinii is transmitted via aerosols, not sexual activity.

Third-trimester fetuses (choice H) are not susceptible to infection with this fungus, although they may become infected in utero with HIV.

A 16-year-old high school cheerleader presents with low grade fever, pleuritic pain and a non-productive cough.

Page 10: Microbiology II

Her serum agglutinates Streptococcus salivarius strain MG. Therapy should include which of the following?

A. Ampicillin

B. Erythromycin

C. Oxygen and external cooling

D. Penicillin G

E. Ribavirin

Explanation:

The correct answer is B. The patient has primary atypical pneumonia caused by Mycoplasma pneumoniae. These organisms are fastidious and difficult to culture in the laboratory, however serodiagnosis can be most helpful. Patients produce one or two heterophile antibodies during the course of the infection: one agglutinates human O+ RBCs in the cold (the cold hemagglutinins) while the other causes the agglutination of a strain of Streptococcus salivarius termed strain MG (the Strep MG agglutinins). Mycoplasma are susceptible to the macrolide family of antibiotics (erythromycin, clarithromycin, etc.). These organisms lack a cell wall, hence they are indifferent to antibiotics that interfere with peptidoglycan synthesis, such as penicillin (choice D) and ampicillin (choice A).

Oxygen and external cooling (choice C) are therapeutic measures that are used in the treatment of severe respiratory diseases such as pneumococcal pneumonia.

Ribavirin (choice E) is used in the treatment of respiratory syncytial virus infection in infants. This is the most common cause of hospitalization for respiratory disease in the very young, probably because aerosol administration of the antiviral compound is best accomplished in a hospital setting.

A 24-year-old summer camp counselor complains of a severe headache and weakness. His condition rapidly deteriorates over a period of hours, and he is airlifted to a nearby hospital. A lumbar puncture is performed and a Gram's stain of spinal fluid reveals gram-negative diplococci. Infection with this organism is also associated with which of the following?

A. Dysentery

B. Erythema chronicum migrans

Page 11: Microbiology II

C. Myocarditis

D. Ophthalmia neonatorum

E. Waterhouse-Friderichsen syndrome

Explanation:

The correct answer is E. The Neisseria are gram-negative diplococci, and have two clinically important species, N. meningitidis and N. gonorrhoeae.N. meningitidis can either cause a sudden, fulminant, life-threatening meningitis or meningococcemia with a vasculitic purpura and disseminated intravascular coagulation. The latter presentation may be complicated by adrenal involvement, precipitating the usually fatal Waterhouse-Friderichsen syndrome with coagulopathy, hypotension, adrenal cortical necrosis, and sepsis. N. gonorrhoeae causes gonorrhea and ophthalmia neonatorum (choice D), a neonatal eye infection.

Classical dysentery (multiple, small-volume stools with blood, mucus, and pus associated with abdominal cramps and tenesmus); (choice A) is caused by Shigella dysenteriae. A similar syndrome can be caused by other bacteria or amoebae as well.

Erythema chronicum migrans (choice B) is the pathognomonic dermatologic manifestation of Lyme disease. This skin lesion consists of an expanding erythematous lesion, with central clearing.

Bacterial myocarditis (choice C) is uncommon, and when it does occur, it usually involves Staphylococcus aureus or Corynebacterium diphtheriae.

A British dairy farmer develops fever with chills, myalgias, headache, skin rash, and vomiting. He is quite ill and is hospitalized. Blood cultures demonstrate tightly coiled, thin, flexible spirochetes shaped like a Shepherd's crook. The spirochetes are easily cultured in serum-enriched nutrient agar. Which of the following organisms should be suspected?

A. Brucella abortus

B. Brucella melitensis

C. Leptospira interrogans

D. Pseudomonas mallei

E. Pseudomonas pseudomallei

Page 12: Microbiology II

Explanation:

The correct answer is C. The only spirochete among the choices is Leptospira interrogans, so even if you didn't know the diseases these organisms produce, you may have been able to answer the question. Leptospirosis, which this patient has, is caused by a spirochete; if you were presented with a list of spirochetes in the choices, the phrase "Shepherd's crook" should tip you off to Leptospira. Clinically, leptospirosis may range from nearly asymptomatic, or at least indistinguishable from other minor flu-like illnesses, to a potentially fatal form (Wal's disease) with jaundice, bleeding, renal failure, and skeletal muscle necrosis. Spread is via contact with blood or urine from infected animals, notably rats. Leptospirosis is found worldwide, but its more severe forms are most likely to occur in the tropics.

Brucella abortus (choice A) is a gram-negative coccus and is one of the causes of brucellosis.

Brucella melitensis(choice B) is a gram-negative coccus and is one of the causes of brucellosis.

Pseudomonas mallei (choice D) and Pseudomonas pseudomallei(choice E) are small gram-negative bacilli that cause melioidosis.

A patient is referred to a neurologist because of ataxia. Neurological examination reveals a loss of proprioception and a wide-based, slapping gate. Magnetic resonance imaging reveals degeneration of the dorsal columns and dorsal roots of the spinal cord. Which of the following organisms is most likely to have caused this pattern of damage?

A. Haemophilus influenzae

B. Herpes simplex I

C. Neisseria gonorrhoeae

D. Neisseria meningitidis

E. Treponema pallidum

Explanation:

The correct answer is E. The findings described are those of tabes dorsalis, a form of tertiary syphilis caused by Treponema pallidum. Tabes dorsalis, and other forms of tertiary syphilis, are now uncommon in this country, possibly because the common use of antibiotics may "treat" many unsuspected cases of syphilis.

Page 13: Microbiology II

Haemophilus influenzae (choice A) and Neisseria meningitidis(choice D) can cause meningitis.

Neisseria gonorrhoeae(choice C) causes gonorrhea, which usually does not involve the CNS.

Herpes simplex I (choice B) can cause an encephalitis that typically involves the frontal and temporal lobes.

A patient presents to a physician with jaundice. Physical examination reveals a nodular, enlarged liver. CT of the abdomen shows a cirrhotic liver with a large mass. CT-guided biopsy of the mass demonstrates a malignant tumor derived from hepatic parenchymal cells. Infection with which of the following viruses would most likely be directly related to the development of this tumor?

A. Epstein-Barr virus (EBV)

B. Hepatitis B virus (HBV)

C. Human herpesvirus type 8 (HHV 8)

D. Human papillomavirus (HPV)

E. Human T-lymphocyte virus (HTLV-1)

Explanation:

The correct answer is B. The tumor is hepatocellular carcinoma, which usually develops in the setting of cirrhosis due to a variety of damaging agents, including hepatitis B virus (HBV) infection, alcohol use, and hemochromatosis.

EBV (choice A) is associated with Burkitt's lymphoma and nasopharyngeal carcinoma.

HHV 8 (a member of the herpes family, choice C) is associated with Kaposi's sarcoma.

HPV (human papillomavirus, choice D) is associated with cervical, penile, and anal carcinoma.

HTLV-1 (human T-lymphocyte virus, choice E) is associated with adult T-cell leukemia.

A 58-year-old alcoholic man with multiple dental caries develops a pulmonary abscess and is treated with antibiotics. Several days later, he develops nausea, vomiting, abdominal pain, and voluminous green diarrhea. Which of the following antibiotics is most likely responsible for this patient's symptoms?

A. Chloramphenicol

Page 14: Microbiology II

B. Clindamycin

C. Gentamicin

D. Metronidazole

E. Vancomycin

Explanation:

The correct answer is B. Any time you see the development of diarrhea in the same question stem as the words "treated with antibiotics," you should immediately think of pseudomembranous colitis. Pseudomembranous colitis is caused by Clostridium difficile and typically occurs as a result of treatment with clindamycin or ampicillin. You would confirm your suspicion by sending a stool culture to be tested for the presence of the C. difficile toxin.

Chloramphenicol's (choice A) most test-worthy side effect is aplastic anemia, not diarrhea. In addition, you might have been able to eliminate this choice simply because of the extremely low probability that this patient would receive this antibiotic in the USA.

Gentamicin's (choice C) key side effects include ototoxicity and nephrotoxicity.

Metronidazole (choice D) and vancomycin (choice E) do not cause pseudomembranous colitis; they are used to treat it.

A 38-year-old AIDS patient presents to the clinic complaining of nausea, occasional vomiting and "bumps" on his groin. On physical examination, multiple, nontender, pedunculated reddish purple nodules in the inguinal and perirectal areas are observed. The patient's liver is palpable 8 cm below the right costal margin. Routine laboratory tests are unremarkable except for an alanine aminotransferase level of 58 and alkaline phosphatase of 90. He denies any foreign travel, but has two pet cats. Which of the following is the most likely cause of this patient's infection?

A. Bartonella henselae

B. Human papillomavirus

C. Molluscum contagiosum virus

D. Rickettsia prowazekii

Page 15: Microbiology II

E. Treponema pallidum

Explanation:

The correct answer is A. Bacillary angiomatosis is a disease that occurs primarily in AIDS patients, and is indicative of a defect in cell-mediated immunity. It is caused by either Bartonella henselae or Bartonella quintana. The domestic cat is the reservoir for these organisms and they are usually transmitted to humans via a cat scratch or cat bite. Patients with this illness usually have multiple skin lesions and extracutaneous manifestations involving liver and bone. Diagnosis is usually based on characteristic histopathologic findings including plump "epithelioid" endothelial cells and mitotic figures. A macrolide, such as erythromycin or azithromycin, is the drug of choice for the infection.

Human papillomavirus (choice B) causes warts. Infection can present as a sessile wart or as condyloma acuminatum, which are fleshy soft growths that coalesce into large masses. When cellular immunity is depressed, as in AIDS, the condylomata acuminatum proliferate.

Molluscum contagiosum virus (choice C) is a pox virus that is spread by close person-to-person contact. Infection produces a firm nodule that often becomes umbilicated, and may resolve by discharging its contents. In AIDS, the lesions do not resolve, but enlarge and spread.

Rickettsia prowazekii(choice D) is the cause of epidemic typhus. It is spread by the human body louse, Pediculus humanis. Its reservoirs are humans and flying squirrels.

Treponema pallidum(choice E) is the spirochete that causes syphilis. The characteristic primary lesion is a chancre (a painless, indurated ulcer) at the site of inoculation.

A 7-year-old girl develops a fever, conjunctivitis, photophobia, and a cough. Her pediatrician notes white spots on a bright red background on the girl's buccal mucosa. Within days, a rash begins around the hairline, then spreads to the trunk and extremities. One week later, the child suddenly begins to convulse, and loses consciousness. She is taken to the emergency room, where involuntary movements and pupillary abnormalities are noted. Which of the following would most likely be seen on CNS biopsy?

A. Demyelination of white matter of cerebral hemispheres with abnormal giant oligodendrocytes

B. Perivenous microglial encephalitis with demyelination

C. Phagocytosis of motor neurons in the spinal cord

Page 16: Microbiology II

D. Severe hemorrhagic and necrotizing encephalitis of the temporal lobe with eosinophilic Cowdry type A inclusion in neurons and glia

E. Small granulomas with central caseation in the meninges

Explanation:

The correct answer is B. The initial history given is classic for measles, with the appearance of Koplik's spots (white spots on the buccal mucosa) followed by a rash beginning along the neck and hairline and spreading to the trunk and extremities. The sequela this child is experiencing is post-infectious encephalomyelitis, which can follow either infection with measles, varicella, rubella, mumps, or influenza, or vaccination with vaccinia vaccine or rabies vaccine derived from nervous tissue. Treatment is supportive, with a mortality of 15 to 40%; survivors frequently have significant permanent neurologic deficits. The pathologic finding is perivenous microglial involvement with demyelination.

Choice A describes the findings of progressive multifocal leukoencephalopathy, a demyelinating disease caused by infection with JC virus, especially in immunocompromised individuals.

Choice C describes the findings of poliomyelitis, a paralytic disease affecting the ventral horn of the spinal cord and motor cortex, caused by an enterovirus (poliovirus).

Choice D describes the findings in herpes encephalitis, which typically affects the inferomedial temporal lobes and orbitofrontal gyri.

Choice E describes the findings in tuberculous meningitis, caused by M. tuberculosis.

A poor African community is experiencing an epidemic of severe hepatitis. The mortality among pregnant women is particularly high. Which of the following viruses is the most likely cause of the epidemic?

A. Cytomegalovirus (CMV)

B. Hepatitis A virus (HAV)

C. Hepatitis C virus (HCV)

D. Herpes simplex I

E. Hepatitis E virus (HEV)

Explanation:

Page 17: Microbiology II

The correct answer is E. Hepatitis E is an important, and until recently, unrecognized cause of epidemics of enterically transmitted acute hepatitis. Hepatitis E is caused by an enterically transmitted virus that occurs primarily in India, Asia, Africa, and Central America. Infection with the virus is associated with a very high mortality among pregnant women.

CMV (choice A) can cause acute hepatitis, but the disease is usually mild and often goes unrecognized, except in profoundly immunosuppressed patients.

Hepatitis A virus (choice B) is the major cause of epidemics of enterically transmitted viral hepatitis, but is not a significant cause of mortality in pregnant women.

Hepatitis C virus (choice C) is usually transmitted parenterally, rather than enterically, and is not a significant cause of mortality in pregnant women.

Herpes simplex (choice D) usually causes significant hepatitis only in profoundly immunosuppressed patients.

A medical worker has a needle-stick accident involving an empty syringe that had been previously used on a patient with a known hepatitis B infection. Which of the following is the most probable outcome for the medical worker?

A. Acute hepatitis followed by recovery

B. "Healthy" carrier

C. Persistent infection followed by recovery

D. Persistent infection progressing to chronic hepatitis

E. Subclinical disease followed by recovery

Explanation:

The correct answer is E. Hepatitis B infection can produce a wide variety of clinical outcomes. The most common outcome (60% to 65%), however, turns out to be subclinical disease followed by complete recovery. The other choices listed show other possible outcomes, and their statistical impact is considered with the discussions of individual choices.

Approximately 20% to 25% of infected persons develop acute hepatitis (choice A), which is followed in 99% of

Page 18: Microbiology II

these cases by recovery and in about 1% of cases by fulminant hepatitis.

Approximately 5% to 10% of cases become "healthy" carriers (choice B).

Approximately 4% of cases develop persistent infection, 67% to 90% of which then recover (choice C) and 10% to 33% of which have chronic hepatitis (choice D).

A 2-day-old baby girl suddenly develops abdominal distention, progressive pallid cyanosis, and irregular respirations. The newborn also has "refused" to breast-feed for the past 18 hours. If the mother was treated for a serious infection with antibiotics for 14 days up to and including the day of delivery, which of the following medications did the mother most likely receive?

A. Aztreonam

B. Chloramphenicol

C. Clindamycin

D. Metronidazole

E. Sulfamethoxazole/trimethoprim

Explanation:

The correct answer is B. Gray "baby" syndrome is a disorder that occurs in newborns who have either received chloramphenicol immediately after birth or whose mothers have received the medication close to the delivery date. Symptoms typically appear in the following order: abdominal distention with or without emesis, progressive pallid cyanosis, and vasomotor collapse, frequently accompanied by irregular respiration. Death can occur as early as a few hours after onset of signs and symptoms. Other symptoms may include: loose, greenish stools, a refusal to suck, ashen color (implied by the name gray baby syndrome), and lactic acidosis. Chloramphenicol is an antimicrobial agent used in the treatment of serious infections when less toxic alternatives are inappropriate.

Aztreonam (choice A) is a beta-lactam antibiotic used primarily in the treatment of gram-negative infections of the urinary tract, lower respiratory tract, and skin, and for intra-abdominal infections. The use of this agent in pregnant or nursing women and infants is considered to be safe and effective.

Clindamycin (choice C) is an anti-infective agent used in the treatment of serious infections when less toxic alternatives are inappropriate. Although the agent is considered to be safe and effective during pregnancy, it is

Page 19: Microbiology II

associated with the development of pseudomembranous colitis and agranulocytosis.

Metronidazole (choice D) is an antibiotic used primarily in the treatment of anaerobic infections. The use of metronidazole should be restricted in pregnancy since newborns have a decreased ability to metabolize this medication. When the elimination of metronidazole is decreased, the severity of adverse reactions increases. Adverse reactions include peripheral neuropathy, seizures, irritability, and profound gastrointestinal disturbances.

Sulfamethoxazole/trimethoprim (choice E) is associated with the development of kernicterus, which is a disorder that can cause abnormal cerebral development in infants. The majority of infants with this disorder generally die within a few weeks of birth. Those infants who survive are often mentally retarded, deaf, or physically impaired.

A 65-year-old man presents with fever, severe headache, and nuchal rigidity. Physical examination in the emergency department shows a Glasgow coma score of 7. Lumbar puncture reveals cloudy cerebrospinal fluid (CSF) with 1200 neutrophils/mm3, elevated protein, and decreased glucose. Which of the following is the most probable etiologic agent of this condition?

A. Arbovirus

B. Herpesvirus

C. Mycobacterium tuberculosis

D. Neisseria meningitidis

E. Streptococcus pneumoniae

Explanation:

The correct answer is E. The clinical manifestations (fever, headache, nuchal rigidity, and low Glasgow coma score), along with the CSF findings (increased neutrophils, elevated protein, and reduced glucose), strongly indicate acute pyogenic (bacterial) meningitis as the underlying condition. Of the microorganisms listed, either Neisseria meningitidis or Streptococcus pneumoniae can cause this form of meningitis; however, Streptococcus pneumoniae is by far the most frequent organism causing acute meningitis in elderly patients.

Arboviruses and herpesviruses (choices A and B) can cause an encephalitis characterized by lymphocytic infiltration of the brain parenchyma and leptomeninges. In this case, CSF findings would include an increased

Page 20: Microbiology II

number of lymphocytes and a normal glucose concentration.

Mycobacterium tuberculosis(choice C) may cause a chronic meningoencephalitis, with a prolonged clinical course. It is characterized pathologically by a dense granulomatous infiltrate of the base of the brain. Associated CSF findings include increased lymphocytes and normal or slightly decreased glucose.

Neisseria meningitidis(choice D) is the classic etiologic agent associated with acute pyogenic meningitis, but it usually affects adolescents and young adults. In neonates, the most frequent organisms include Escherichia coli and group B streptococci; in infants and children, Hemophilus influenzae; and in the elderly, Streptococcus pneumoniae and Listeria monocytogenes.

A 12-year-old boy has a productive cough characterized by large volumes of foul-smelling sputum. Three years ago, the patient was diagnosed with pancreatic insufficiency, as evidenced by repetitive gastrointestinal symptoms of steatorrhea. After culture of the sputum, colorless, oxidase-positive colonies with a fruity aroma develop on the agar. The function of which of the following proteins is most likely inhibited by the bacteria responsible for this boy's infection?

A. A CFTR protein lacking a phenylalanine in exon 10 at position 508

B. A GTP-binding protein involved in the elongation step of protein synthesis

C. A GTP-binding protein similar to the one coupled with α2-adrenergic receptors

D. A GTP-binding protein similar to the one coupled with β-adrenergic receptors

E. A phosphorylation-regulated chloride channel in the apical membrane of epithelial cells

Explanation:

The correct answer is B. The boy is displaying the characteristic symptoms of cystic fibrosis with bronchiectasis (accounting for the foul-smelling sputum) and pancreatic insufficiency, producing steatorrhea. However, the question directly relates to the infectious agent causing the pneumonia, Pseudomonas aeruginosa, an oxidase-positive bacteria with a fruity aroma. P. aeruginosa is a strictly aerobic gram-negative rod that produces an exotoxin (exotoxin A) that ADP-ribosylates, and therefore inhibits, eukaryotic elongation factor 2 (eEF-2), which is a G-protein involved in the eukaryotic translation of proteins.

The cystic fibrosis transmembrane conduction regulator (CFTR) protein (choice A) is the product of the cystic

Page 21: Microbiology II

fibrosis gene, a large 24 exon gene located on the long arm of chromosome 7 (7q31). This protein has two nucleotide binding domains. In 70% of the families with the disease, a three base pair deletion at codon 508 of exon 10 results in the deletion of a phenylalanine (DF508). This codon is critical for one of the nucleotide binding sites of the CF gene product, resulting in poor function of a phosphorylation-regulated chloride ion channel in the apical membrane of epithelial cells. The defective chloride channel leads to impaired secretion of chloride in the lumen and promotes the absorption of sodium inside the cells. Water follows, concentrating the airway secretions. The increased viscosity of the airway secretions predisposes the patient to repeated infections.

A GTP-binding protein similar to the one coupled with α2-adrenergic receptors (choice C) refers to a G-protein that inhibits adenylate cyclase, lowering cAMP. This G-protein is sensitive to Pertussis toxin.

A GTP-binding protein similar to the one coupled with β-adrenergic receptors (choice D) refers to a G-protein that stimulates adenylate cyclase, increasing cAMP. This G-protein is sensitive to cholera toxin.

The product of the cystic fibrosis gene is a phosphorylation-regulated chloride channel in the apical membrane of epithelial cells (choice E).

During the asymptomatic latent phase of AIDS, the virus is actively proliferating, and can be found in association with

A. B lymphocytes

B. follicular dendritic cells in lymph nodes

C. ganglion cells

D. oligodendrocytes

E. peripheral nerves

Explanation:

The correct answer is B. Follicular dendritic cells in the germinal centers of lymph nodes are important reservoirs of HIV. Although some follicular dendritic cells are infected with HIV, most viral particles are found on the surface of their dendritic processes. Follicular dendritic cells have receptors to the Fc portion of immunoglobulins that serve to trap HIV virions coated with anti-HIV antibodies. These coated HIV particles retain

Page 22: Microbiology II

the ability to infect CD4+ T cells as they traverse the dendritic cells.

B lymphocytes (choice A) have a surface marker (CD21 protein-a complement receptor) to which an Epstein-Barr envelope glycoprotein can bind. The virus associates with the host cell genome, producing a latent infection. These B cells undergo polyclonal activation and proliferation.

Ganglion cells (choice C), particularly the satellite cells around the ganglion cells in the dorsal root ganglia, can be infected by varicella-zoster. Herpes type I and II infect neurons that innervate skin and mucous membranes.

Oligodendrocytes (choice D) are directly infected by two viruses, JC virus (a polyomavirus) and measles virus. JC virus causes progressive multifocal leukoencephalopathy (PML), and measles virus produces a latent syndrome called subacute sclerosing panencephalitis (SSPE).

Peripheral nerves (choice E) are indirectly affected by HIV virus in the AIDS-associated myopathy. The disease is characterized by a subacute onset of proximal muscle weakness, sometimes with pain, and elevated levels of creatine kinase. The muscles and nerves are infiltrated with mononuclear cells, including HIV-positive macrophages.

A viral organism was isolated from a painful blister on the lip of a teenage girl. The agent was found to double-stranded, linear DNA and was enveloped. The patient had a similar sore approximately 2 months ago. Which of the following is the most likely causative organism?

A. Adenovirus

B. Coxsackie virus

C. Herpes simplex type 1 virus

D. Herpes zoster virus

E. Papilloma virus

Explanation:

The correct answer is C. Herpes simplex is an enveloped, linear DNA virus that is a very common infectious agent; most adults will have anti-Herpes simplex antibodies in their serum, although many may not have ever had any clinical signs of disease. The hallmark of this disease is painful skin vesicles often called "cold" sores, or "fever" blisters to denote the precipitating event that preceded the appearance of the lesions. The virus has

Page 23: Microbiology II

a propensity to become latent in the host, finding safe refuge in nervous tissue. Activation of the infection occurs following mild trauma (e.g., a visit to the dentist), hormonal changes (e.g., menses), and immunosuppression (e.g., following organ transplantation). Other, more serious, manifestations of disease include encephalitis, pneumonia, and hepatitis; these are particularly likely to be seen in immunodeficient patients such as those with AIDS.

Adenoviruses (choice A) are naked, linear, double-stranded DNA viruses that cause acute, usually self-limiting, influenza-like illnesses occurring in the fall and winter. The symptoms include pharyngitis, fever, cough, and general malaise. Epidemic pharyngoconjunctivitis and pneumonia can occur in closed populations such as military installations.

Coxsackie viruses (choice B) are naked, single-stranded, polycistronic viruses with an RNA genome. They are divided into groups A and B based on their virulence in suckling mice. Coxsackie A group causes generalized myositis and flaccid paralysis, which is rapidly fatal to infant mice, whereas group B produces less severe lesions of the heart, pancreas, and central nervous system. In man, Coxsackie A causes herpangina and hand-foot-and-mouth disease, while Coxsackie B is seen in patients with pleurodynia, myocarditis, and pericarditis. Both groups cause upper respiratory infections, febrile rashes, and meningitis.

Herpes zoster (choice D), the varicella virus, is an enveloped, double-stranded DNA virus that is a very common infectious agent in children. Chickenpox is a mild, self-limiting illness in children that is evidenced as a fever followed by a macular rash that progresses to papules, then vesicles of the skin and mucous membranes. Shingles is a recurrence of a previously latent varicella infection in which the virus has taken refuge in sensory ganglia of spinal or cranial nerves. Various factors that decrease the immune status of the patient contribute to the exacerbation of the infection. Severe dermatomal pain occurs with a vesicular eruption, fever, and malaise.

Papilloma viruses (choice E) are members of the Papovavirus family. They are non-enveloped and possess a double-stranded, circular DNA genome. They cause skin, plantar, and genital warts; some serotypes of human papilloma viruses (e.g., HPV-16) are associated with penile, laryngeal, and cervical carcinomas.

A viral organism was isolated from a painful blister on the lip of a teenage girl. The agent was found to double-stranded, linear DNA and was enveloped. The patient had a similar sore approximately 2 months ago. Which of the following is the most likely causative organism?

A. Adenovirus

Page 24: Microbiology II

B. Coxsackie virus

C. Herpes simplex type 1 virus

D. Herpes zoster virus

E. Papilloma virus

Explanation:

The correct answer is C. Herpes simplex is an enveloped, linear DNA virus that is a very common infectious agent; most adults will have anti-Herpes simplex antibodies in their serum, although many may not have ever had any clinical signs of disease. The hallmark of this disease is painful skin vesicles often called "cold" sores, or "fever" blisters to denote the precipitating event that preceded the appearance of the lesions. The virus has a propensity to become latent in the host, finding safe refuge in nervous tissue. Activation of the infection occurs following mild trauma (e.g., a visit to the dentist), hormonal changes (e.g., menses), and immunosuppression (e.g., following organ transplantation). Other, more serious, manifestations of disease include encephalitis, pneumonia, and hepatitis; these are particularly likely to be seen in immunodeficient patients such as those with AIDS.

Adenoviruses (choice A) are naked, linear, double-stranded DNA viruses that cause acute, usually self-limiting, influenza-like illnesses occurring in the fall and winter. The symptoms include pharyngitis, fever, cough, and general malaise. Epidemic pharyngoconjunctivitis and pneumonia can occur in closed populations such as military installations.

Coxsackie viruses (choice B) are naked, single-stranded, polycistronic viruses with an RNA genome. They are divided into groups A and B based on their virulence in suckling mice. Coxsackie A group causes generalized myositis and flaccid paralysis, which is rapidly fatal to infant mice, whereas group B produces less severe lesions of the heart, pancreas, and central nervous system. In man, Coxsackie A causes herpangina and hand-foot-and-mouth disease, while Coxsackie B is seen in patients with pleurodynia, myocarditis, and pericarditis. Both groups cause upper respiratory infections, febrile rashes, and meningitis.

Herpes zoster (choice D), the varicella virus, is an enveloped, double-stranded DNA virus that is a very common infectious agent in children. Chickenpox is a mild, self-limiting illness in children that is evidenced as a fever followed by a macular rash that progresses to papules, then vesicles of the skin and mucous membranes.

Page 25: Microbiology II

Shingles is a recurrence of a previously latent varicella infection in which the virus has taken refuge in sensory ganglia of spinal or cranial nerves. Various factors that decrease the immune status of the patient contribute to the exacerbation of the infection. Severe dermatomal pain occurs with a vesicular eruption, fever, and malaise.

Papilloma viruses (choice E) are members of the Papovavirus family. They are non-enveloped and possess a double-stranded, circular DNA genome. They cause skin, plantar, and genital warts; some serotypes of human papilloma viruses (e.g., HPV-16) are associated with penile, laryngeal, and cervical carcinomas.

Which of the following is a feature of gram-positive bacteria rather than gram-negative bacteria?

A. Lipid A-containing lipopolysaccharide

B. Lipoprotein in periplasmic space

C. Outer membrane

D. Peptidoglycan in periplasmic space

E. Thick peptidoglycan cell wall

Explanation:

The correct answer is E. Most of the features listed are those of gram-negative bacteria, which have a complex cell envelope consisting of a cytoplasmic (inner) membrane, a periplasmic space containing peptidoglycan (choice D) and lipoprotein (choice B), an outer membrane (choice C), and sometimes a capsule. The outer membrane contains lipopolysaccharide (choice A) which is a major component of endotoxin. The peptidoglycan cell wall of the gram-negative bacteria is thin, while that of the gram-positive bacteria is thick. Other features of gram- positive bacteria include a fairly simple surface with cytoplasmic membrane, peptidoglycan, cell wall, and sometimes, an outer capsule. The cell wall contains lipoteichoic acids.

A mailman gets a severe bite wound from a pit bull guarding a junkyard. The wound is cleansed and he receives a booster injection of tetanus toxoid and an injection of penicillin G. Several days later, the wound is inflamed and purulent. The exudate is cultured on blood agar and yields gram-negative rods. Antibiotic sensitivity tests are pending. The most likely agent to be isolated is

A. Bartonella henselae

Page 26: Microbiology II

B. Brucella canis

C. Clostridium tetani

D. Pasteurella multocida

E. Toxocara canis

Explanation:

The correct answer is D. Pasteurella multocida is a gram-negative rod that is normal flora of the oral cavity of dogs and cats. It often causes a local abscess following introduction under the skin by an animal bite. Most cases occur in children who are injured while playing with a pet.

Bartonella henselae (choice A) is a very small, gram-negative bacterium that is closely related to the rickettsia, although it is able to grow on lifeless media. It is the cause of cat-scratch disease (a local, chronic lymphadenitis most commonly seen in children) and bacillary angiomatosis (seen particularly in AIDS patients). In this latter patient population, the organism causes proliferation of blood and lymphatic vessels causing a characteristic "mulberry" lesion in the skin and subcutaneous tissues of the afflicted individual.

Brucella canis(choice B) is a gram-negative rod that is a zoonotic agent. Its normal host is the dog, but when it gains access to humans, it causes an undulating febrile disease with malaise, lymphadenopathy and hepatosplenomegaly. The normal route of exposure is via ingestion of the organism.

Clostridium tetani(choice C) is a gram-positive spore-forming anaerobic rod. It causes tetanus (a spastic paralysis caused by tetanospasmin, which blocks the release of the inhibitory neurotransmitters glycine and gamma-aminobutyric acid [GABA]). There may be no lesion at the site of inoculation, and exudation would be extremely rare.

Toxocara canis (choice E), a common intestinal parasite of dogs, is a metazoan parasite that causes visceral larva migrans. Young children are most likely to be affected, as they are most likely to ingest soil contaminated with eggs of the parasite.

The World Health Organization identifies an alarming increase in hospital admissions worldwide attributable to a new and unexpected serotype of influenza A virus. The biological attribute of influenza A virus, which allows the sudden appearance of dramatically new genetic variants, is also present in a limited number of other viral families. Which of the following viruses also possesses this biological attribute?

Page 27: Microbiology II

A. Coronavirus

B. HIV

C. Measles virus

D. Rabies virus

E. Rotavirus

F. Rubella virus

G. St. Louis encephalitis virus

Explanation:

The correct answer is E. This case describes a pandemic of influenza A, which is caused by the ability of the virus to undergo dramatic genetic changes of type by reassortment of its segmented RNA genome - a trait called genetic shift. The only virus on the list that possesses a segmented genome is the rotavirus, in the reovirus family, which possesses 10-11 segments in its genome.

Coronavirus (choice A) is not segmented and is a cause of the common cold.

HIV (choice B) is not segmented and is known for its genetic drift (minor mutational changes over time due to an error-prone polymerase), not genetic shift.

Measles virus (choice C) is not segmented and is controlled largely by vaccination.

Rabies virus (choice D) is not segmented.

Rubella virus (choice F) is not segmented.

St. Louis encephalitis virus (choice G) is a flavivirus and is not segmented.

A 35-year-old woman presents to her gynecologist because of vaginal itchiness and discharge. Pelvic examination demonstrates abundant white, curdy material in the vagina. Microscopic examination of the material demonstrates fungal hyphae and yeast forms. Which of the following systemic diseases can predispose for this condition?

A. Crohn's disease

B. Diabetes mellitus

Page 28: Microbiology II

C. Disseminated gonococcal infection

D. Rheumatoid arthritis

E. Systemic lupus erythematosus

Explanation:

The correct answer is B. The patient has vulvovaginitis secondary to Candida infection. Predisposing factors include a high vaginal pH, diabetes, and use of antibiotics. The increased vulnerability in diabetes may reflect increased glucose concentrations in vaginal secretions and relative immunosuppression.

Crohn's disease (choice A) can predispose for fistulas involving the vagina, rather than vulvovaginitis.

Dissemination of Neisseria gonorrhoeae(choice C) can cause septic arthritis.

Neither rheumatoid arthritis (choice D) nor systemic lupus erythematosus (choice E) are specifically associated with Candida vulvovaginitis.

A 1-week-old female infant with symptoms of vomiting and anorexia has a temperature of 102° F. A bulging fontanel is noted on physical examination. The most likely agent is

A. Haemophilus influenzae type b

B. Listeria monocytogenes

C. Neisseria meningitidis

D. Staphylococcus aureus

E. Streptococcus agalactiae

Explanation:

The correct answer is E. Vomiting, anorexia, high fever (above 100.4° F), and a bulging fontanel equals neonatal meningitis until proven otherwise. Streptococcus agalactiae (group B strep) and Escherichia coli (not an answer choice) are the most common causes in neonates up to 1 month of age. The next most reasonable response would have been Listeria monocytogenes(choice B), another, though less common, cause of neonatal meningitis.

Most cases of meningitis caused by Haemophilus influenzae(choice A) occur in children between 6 months and

Page 29: Microbiology II

6 years of age, 90% of which result from the capsular type b strain. It has become much less prevalent since the H. influenzae type b conjugate vaccine has been routinely administered to infants.

Neisseria meningitidis(choice C) is the most common cause of epidemic meningitis. The two organisms most often associated with sporadic cases are Haemophilus influenzae and Streptococcus pneumoniae (the most common cause in adults over 30).

Staphylococcus aureus(choice D) is not a common cause of meningitis, except in patients with CSF shunts. It is often responsible for abscesses, osteomyelitis, endocarditis, toxic shock syndrome, and food poisoning.

A 7-month-old child presents with a 4-day history of fever, deepening cough, and dyspnea. A chest x-ray shows multiple interstitial infiltrates and hyperinflation of the lungs. Multinucleated giant cells with cytoplasmic inclusion bodies are seen when a nasal wash is inoculated into culture. The most appropriate therapy includes administration of which of the following drugs?

A. Acyclovir

B. Ganciclovir

C. Ribavirin

D. Trifluorothymidine

E. Zidovudine

Explanation:

The correct answer is C. Ribavirin is an antiviral drug approved for the treatment of severe respiratory syncytial virus infection, the most common cause of pneumonia and bronchiolitis in children under 1 year of age. It should be given by aerosol.

Acyclovir (choice A) is a guanosine analog that is useful for the treatment of primary and recurrent herpes infections and herpes simplex virus encephalitis.

Ganciclovir (choice B) is a guanosine analog used in the treatment of cytomegalovirus retinitis and cytomegalovirus infections in AIDS patients.

Trifluorothymidine (choice D) is a thymidine analog used topically for the treatment of recurrent epithelial keratitis and primary keratoconjunctivitis due to herpes simplex viruses.

Page 30: Microbiology II

Zidovudine (choice E) is a thymidine analog that inhibits reverse transcriptase. It is active against human retroviruses, including HIV-1, HIV-2, and HTLV-1.

A 6-year-old boy presents to the pediatric clinic with fever and earache. He has just finished an unsuccessful course of amoxicillin. On physical exam, his right tympanic membrane appears injected. Which of the following antimicrobials would be most appropriate to prescribe for this patient?

A. Amphotericin B

B. Bacitracin

C. Cefaclor

D. Erythromycin

E. Sulfamethoxazole

Explanation:

The correct answer is C. The drug of choice for otitis media in children is amoxicillin. But in refractory cases, often due to bacterial resistance, switching to a different drug class is often effective. You must look for another medication that is effective against common organisms responsible for pediatric otitis media, such as Streptococcus pneumoniae (a gram-positive diplococcus) and Haemophilus influenzae (a gram-negative rod). A second-generation cephalosporin, such as cefaclor, should cover both and is the best choice. Consequently, it is commonly used in cases of amoxicillin-resistant otitis media. None of the other choices cover the proper spectrum of organisms.

Amphotericin B (choice A) is an antifungal polyene. It works by binding to ergosterol in the fungal cell membrane, creating an artificial pore. It is used to treat systemic mycoses such as Aspergillus, Blastomyces, Candida, Coccidioides, Cryptococcus, and Histoplasma.

Bacitracin (choice B) is a topical agent used to fight infection with gram-positive organisms. It interferes with cell wall synthesis.

Erythromycin (choice D) is a macrolide antibiotic that binds to the 23s rRNA portion of the 50s subunit of ribosomes, inhibiting release of uncharged tRNA and stopping protein synthesis. Though effective against S. pneumoniae, it is not particularly active against H. influenzae. Note that erythromycin may be used in amoxicillin-resistant otitis media, but only when administered with a sulfonamide such as sulfisoxazole.

Page 31: Microbiology II

Sulfamethoxazole (choice E) is a sulfonamide. It is bacteriostatic and works by inhibiting folic acid synthesis. It resembles p-aminobenzoic acid (PABA) structurally. When combined with trimethoprim (a dihydrofolate reductase inhibitor) it exerts a bactericidal effect and serves as the drug combination of choice for complicated urinary tract infections.

A previously healthy 18-month-old girl is brought to the office with 2 days of irritability, poor appetite, and pulling at her left ear. She has no known allergies, and her vaccinations are up-to-date. On examination, the child's temperature is 102.8 F. She is easily consoled by the mother and moves her neck spontaneously without discomfort. There is a clear discharge from the nares. The left tympanic membrane is erythematous, dull, and bulging. Which of the following virulence factors is generally absent in the strains of the causative organism that produce otitis media, compared with those that produce epiglottitis or meningitis?

A. β-Lactamase

B. IgA protease

C. Lipopolysaccharide

D. Nonpilus adhesins

E. Pili

F. Pneumolysin

G. Polyribitol phosphate

Explanation:

The correct answer is G. This is most likely a case of Haemophilus influenzae otitis media. 95% of all cases of invasive disease (epiglottitis, meningitis) due to H. influenzae are caused by type b organisms that possess a polyribitol phosphate capsule. Otitis media is generally not caused by type b organisms.

β-Lactamase (choice A) is an important pathogenic feature of Moraxella catarrhalis, which is another important cause of otitis media, but would not be an agent of epiglottitis or meningitis.

IgA protease (choice B) is produced by Streptococcus pneumoniae and Neisseria meningitidis. Both of these cause meningitis, but not as commonly in this age group, and would not be the most common causes of otitis media in this case.

Page 32: Microbiology II

Lipopolysaccharide (choice C) is present in all gram-negative bacteria and would not be a distinguishing feature between those that cause otitis media and epiglottitis.

Nonpilus adhesins (choice D) are mediators of attachment to the epithelium and colonization of the oropharynx, but would not be a primary difference between the agents of otitis media and epiglottitis.

Pili (choice E) also mediate attachment to the oropharynx, but would not be the major difference between the agents of otitis media and epiglottitis.

Pneumolysin (choice F) is a cytotoxin produced by S. pneumoniae that destroys ciliated epithelial cells.

Which of the following organisms is most likely to be implicated as a cause of urethritis that persists after antibiotic therapy for gonorrhea?

A. Actinomyces

B. Chlamydia

C. Mycobacteria

D. Nocardia

E. Rickettsia

Explanation:

The correct answer is B.Chlamydia, Mycoplasma, and Ureaplasma are not effectively treated by penicillins and cephalosporins, and are important causes of post-gonococcal urethritis. Chlamydial urethritis can be diagnosed by using fluorescent antibodies to identify inclusions in epithelial cells.

Actinomyces(choice A) is a mouth commensal that rarely causes a deeper oral infection.

Mycobacteria(choice C) cause chronic granulomatous diseases such as tuberculosis and leprosy.

Nocardia(choice D) can cause necrotizing pneumonia and disseminated disease.

Rickettsia(choice E) cause typhus and Rocky Mountain spotted fever.

Which of the following is more frequently associated with Klebsiella pneumoniae than with Pseudomonas aeruginosa?

A. Artificial ventilation

Page 33: Microbiology II

B. Cystic fibrosis

C. Diabetes mellitus

D. Green-colored sputum

E. Upper lobe cavitation

Explanation:

The correct answer is E.Klebsiella pneumoniae is a well-recognized cause of community-acquired lobar pneumonia associated with cavitation. It is found typically in alcoholic males over 40 years of age with underlying diabetes or obstructive lung disease. Klebsiella pneumoniae mimics Streptococcus pneumoniae as a pulmonary pathogen except that Klebsiella has a greater tendency to progress to lung abscess and empyema. Pseudomonas aeruginosa is usually associated with patients on ventilators, particularly in intensive care units. Immunocompetent patients usually have bilateral bronchopneumonia without cavitary lesions.

Artificial ventilation (choice A) is classically associated with P. aeruginosa infection. The organism thrives in a wet environment such as respirators, cleaning solutions, disinfectants, sinks, vegetables, flowers, endoscopes, and physiotherapy pools.

P. aeruginosa is a very important pathogen. Mucoid strains of this organism infect the airways in patients with cystic fibrosis (choice B), leading to acute exacerbations and chronic progression of lung damage.

Both organisms cause disease in association with diabetes (choice C). Klebsiella pneumoniae produces pulmonary disease and P. aeruginosa causes necrotic skin ulcers in diabetics.

Green-colored sputum (choice D) is associated with P. aeruginosa, since more than half of the clinical isolates produce the blue-green pigment pyocyanin, which is helpful in identifying the organism.

A 23-year-old woman presents to the emergency room with pelvic pain. A Gram's stain of her cervical discharge reveals multiple polymorphonuclear leukocytes, but none contain gram-negative diplococci. Which of the following statements best describes the two organisms that most commonly cause this disorder?

A. Both are unlikely to recur because of acquired cell mediated immunity

B. Both are unlikely to recur because of antibody mediated immunity

C. Both induce endocytosis by epithelial cells

Page 34: Microbiology II

D. Both are obligate intracellular parasites

E. Both respond to β-lactam antibiotics

Explanation:

The correct answer is C. In young women, the most likely causes of cervicitis and pelvic inflammatory disease (PID) are Neisseria gonorrhoeae and Chlamydia trachomatis. Gram's staining alone may not be able to distinguish between the two in women, so culture is warranted. Both organisms induce endocytosis by epithelial cells.

Neisseria gonorrhoeae is a gram-negative diplococcus. It is endocytosed by mucus-secreting epithelia, and is exocytosed into subepithelial tissues leading to necrotizing acute inflammation and destruction of ciliated and non-ciliated cells. Through direct extension, it can lead to PID and infertility in women. It can be detected by visualizing the organism in polymorphonuclear leukocytes (PMNs) in Gram's stained clinical material more easily in men than in women. Therefore, culture onto Thayer-Martin agar is usually required for diagnosis in women. Since the organism can be cultured, it is not an obligate intracellular parasite (compare with choice D). The organism can be killed by PMNs, antibody, and complement, however, many of its outer membrane proteins undergo antigenic or phase variation or cause the production of blocking antibody that interferes with bactericidal activity. Immunity to the organism does not develop (compare with choices A and B). Neisseria gonorrhoeae is resistant to penicillin, a β-lactam antibiotic (compare to choice E), because of a plasmid-mediated penicillinase and a chromosomally-mediated mutation that decreases the permeability of the outer membrane and decreases the affinity of penicillin binding proteins. It can be treated with ceftriaxone, a cephalosporin, which resists the penicillinase, or with quinolones and azithromycin.

Chlamydia trachomatis is an obligate intracellular parasite (choice D) that induces endocytosis and resides in the phagosome of infected epithelial cells. It possesses a gram-negative envelope but lacks peptidoglycan, and hence is intrinsically resistant to all antibiotics that inhibit peptidoglycan synthesis, such as the β-lactam antibiotics (compare with choice E). Antibodies are ineffective at controlling infection. Cell mediated immunity (choice A) is probably the major means of controlling this infection since a deficiency in cell mediated immunity both increases susceptibility and severity in animal models. Chronic and clinically latent infections cause serious morbidity, including pelvic inflammatory disease, fallopian tube dysfunction and blindness. Chlamydia infection can be treated with macrolides, quinolones and tetracyclines.

Page 35: Microbiology II

While playing in the park, an 8-year-old girl is bitten in the leg by a neighbor's cat. She presents the next day with fever and bone pain localized to her right calf. X-ray reveals a lytic lesion of the right tibia. Results of the bone culture are pending. You expect the infecting organism to be

A. Brucella melitensis

B. Eikenella corrodens

C. Francisella tularensis

D. Pasteurella multocida

E. Yersinia pestis

Explanation:

The correct answer is D. This patient has osteomyelitis due to a cat bite that penetrated the periosteum. Whenever you see dog or cat bites in a question stem, consider Pasteurella multocida as a primary cause of wound infection. This organism is a short, encapsulated, gram-negative rod demonstrating bipolar staining. Rapidly arising cellulitis is particularly indicative of this organism.

Brucella melitensis(choice A) is a small, acapsular, gram-negative rod that causes brucellosis (undulant fever) and is associated with contact with goats or sheep. Brucella abortus and Brucella suis are variants associated with contact with cows and pigs, respectively. The organisms enter the body through the skin or through contaminated dairy products, such as unpasteurized imported goat's milk or cheeses.

Eikenella corrodens(choice B) is commonly found in human bites. It is a gram-negative rod that is part of the normal flora of the human mouth.

Francisella tularensis(choice C) is a small, pleomorphic, gram-negative rod that causes tularemia (rabbit fever). It occurs most commonly in rural areas. In the USA, rabbits are the main reservoir for this organism, which is transmitted to humans by the Dermacentor tick or by contact with infectious animal tissues.

Yersinia pestis(choice E) is responsible for bubonic plague, which has been known to occur in the western USA. Its main reservoir is the prairie dog, and its vector is the rat flea.

A 6-week-old infant is brought to the emergency room with a 10-day history of coughing and choking spells. The

Page 36: Microbiology II

white blood cell count is elevated with 80% lymphocytes. The child is gasping for breath, experiencing paroxysms of coughing, and vomits twice in the examination room. Encapsulated, gram-negative rods grow out on Bordet-Gengou media. Which of the following microorganisms is the likely cause of these symptoms?

A. Bordetella pertussis

B. Haemophilus influenzae type b

C. Klebsiella pneumoniae

D. Legionella pneumophila

E. Mycoplasma pneumoniae

Explanation:

The correct answer is A. The child has whooping cough, or more accurately if one goes by the chronology of the symptoms, "coughing whoop," as the patient is subject to a series of coughing episodes that are followed by a rapid inspiration of air; the "whoop" is caused by the rapid passage of air through a narrowed airway. The infection of the respiratory tract by Bordetella pertussis causes a hyperreactivity of the respiratory apparatus; even the slightest stimulus will trigger a coughing episode. The offending pathogen excretes adenylate cyclase and also produces an exotoxin that inactivates the inhibitory subunit of the G-protein complex, thus activating adenylate cyclase within the cells of the respiratory system. Bordet-Gengou agar is the blood-enriched (15% vs. 5% for normal blood agar) medium used for the laboratory isolation of this pathogen. Fluorescent antibody testing is used to verify the diagnosis.

Haemophilus influenzae type b (choice B) was the major cause of infant meningitis before the Hib conjugate vaccine nearly eradicated this pathogen from the United States. Nonencapsulated strains cause otitis media in children and pneumonia in adults. This organism also has a peculiar growth requirement, requiring factor X (hematin) and factor V (NAD).

Klebsiella pneumoniae(choice C) is a gram-negative, highly encapsulated rod that is a significant pulmonary pathogen in individuals with a compromised respiratory apparatus. It is a common cause of aspiration pneumonia and pulmonary abscesses in alcoholics and patients with chronic obstructive pulmonary disease. The organism is readily grown on standard laboratory media such as blood agar or MacConkey's enteric agar.

Legionella pneumophila(choice D) is another fastidious, gram-negative, respiratory pathogen that may cause

Page 37: Microbiology II

either a fulminating disease or a mild "walking pneumonia-like" condition (i.e., an atypical pneumonia). The organism can be cultured on a charcoal yeast extract medium, but identification is usually accomplished by immunofluorescent examination of the pulmonary specimen.

Mycoplasma pneumoniae(choice E) is the most common cause of primary atypical pneumonia. The disease is considered "atypical" because the patients have a very mild disease with low-grade fever, little in the way of constitutional signs, and a non-productive cough. These organisms are fastidious and are difficult to grow in the laboratory. Diagnosis is by immunofluorescent staining or by demonstration of cold hemagglutinins and strep MG agglutinins in the serum of the patient.

A patient is admitted to a psychiatric hospital after having been picked up by the police for making inappropriate sexual advances. A detailed psychiatric interview demonstrates deficits in memory, insight, judgement, personal appearance, and social behavior. The patient is witnessed experiencing a possible epileptic seizure. Over a period of several years, motor findings also develop, including relaxed, but expressionless facies, tremor, dysarthria, and pupillary abnormalities. Which of the following tests performed on his cerebrospinal fluid would most likely be diagnostic?

A. CSF glucose

B. FTA-ABS

C. Gram's stain

D. Lymphocyte count

E. Neutrophil count

Explanation:

The correct answer is B. The patient has neurosyphilis, specifically general paresis, a term that means "general paralysis of the insane." In this late sequela of syphilitic infection which occurs 5 to 20 years after infection, patients develop mental deterioration, which precedes motor system deterioration, leading eventually to "general paralysis" with mutism and incontinence. The abnormalities may be conveniently recalled using paresis as a mnemonic for personality, affect, hyperactive reflexes, Argyll Robertson pupils in the eyes, defects

Page 38: Microbiology II

in the sensorium, intellectual decline and deficient speech. Specific anti-treponemal tests such as FTA-ABS are usually positive on both serum and cerebrospinal fluid.

The cerebrospinal (CSF) glucose (choice A) in neurosyphilis is usually normal.

Gram's stain (choice C) of CSF will not demonstrate spirochetes in neurosyphilis.

The CSF lymphocyte count (choice D) is typically elevated in neurosyphilis, but this is a non-specific finding.

The CSF neutrophil count (choice E) is usually normal in neurosyphilis.

A 65-year-old man develops a vesicular rash localized to a narrow circumferential band on one side of his chest. The rash is very painful, and the vesicles are confluent with some ulceration. No other significant findings are demonstrated on physical examination. Which of the following diagnoses is most likely to be correct?

A. Chicken pox

B. Herpes simplex I infection

C. Herpes simplex II infection

D. Measles

E. Shingles

Explanation:

The correct answer is E. This is shingles, the recurrent form of herpes zoster infection, which is usually (except in the case of immunosuppressed patients) confined to a single dermatome. Isolated vesicles may be seen outside the dermatome. The primary herpes zoster infection precedes the development of shingles by years or decades; the prevalence of shingles rises steadily with age, to the point that 1% of people older than 80 years have the condition. Shingles lesions are infections and should be considered an infectious hazard in the hospital setting. Acyclovir can ameliorate the condition.

Varicella, or chicken pox (choice A), is the primary form of herpes zoster infection and affects face and trunk diffusely.

Herpes simplex I (choice B) affects oral and perioral sites.

Herpes simplex II (choice C) primarily affects genital sites.

Measles (choice D) causes a blotchy rash, rather than a dermatomal one.

Page 39: Microbiology II

The electron transport system of Neisseria is located on which of the following structures?

A. Cytoplasmic membrane

B. Mesosome

C. Mitochondria

D. Nuclear membrane

E. Polyribosome DNA aggregates

Explanation:

The correct answer is A. Unlike eukaryotic cells in which the electron transport system is located on mitochondria, the electron transport system of bacteria is located on the cytoplasmic (plasma) membrane.

Mesosomes (choice B) and polyribosome DNA aggregates (choice E) are also found in bacterial cells and function in cell division and protein synthesis, respectively.

Bacteria do not have mitochondria (choice C) or membrane-bounded nuclei (choice D).

A Pap smear from a 30-year-old woman demonstrates protozoal parasites. These organisms are likely to be

A. Cryptosporidium parvum

B. Entamoeba histolytica

C. Giardia lamblia

D. Isospora belli

E. Trichomonas vaginalis

Explanation:

The correct answer is E.Trichomonas vaginalis (note "vagina" in the name) is the only organism listed to primarily affect the genital tract rather than the intestinal tract. The vaginitis it causes is characterized by a frothy, yellow discharge. The organism can be identified in Pap smears, or, more reliably, by special culture techniques that are becoming more widely available. Infection in men is usually asymptomatic, but prostatitis and urethritis can also occur. Treatment with metronidazole is indicated for both the patient and the sexual partner(s).

Page 40: Microbiology II

The other organisms listed (choices A, B, C, and D) cause gastrointestinal disease.

A previously healthy European child who had been diving into a lake develops meningoencephalitis. The disease is rapidly fatal, despite aggressive medical therapy. Which of the following organisms is most likely to be responsible for this child's illness?

A. Acanthamoeba sp.

B. Balantidium coli

C. Entamoeba histolytica

D. Giardia lamblia

E. Naegleria fowleri

Explanation:

The correct answer is E. Two free-living amoeba have a specific propensity for causing meningoencephalitis: Naegleria fowleri and Acanthamoeba sp. The former is seen in the clinical setting described in the question stem, and the infection appears to develop when amoebae in lake water are forced across the cribriform plate in the nose during swimming and particularly diving. Since the olfactory bulbs are actually part of the brain, crossing the cribriform plate allows the amoeba access to the meninges and brain. Diagnosis can be difficult because the causative organism may resemble human cells when seen in cerebrospinal fluid. Unfortunately, treatment is also difficult because the organism is not very sensitive to presently available anti-protozoan drugs.

Acanthamoeba sp. (choice A) can also cause meningoencephalitis, but the usual setting is in immunosuppressed patients.

Balantidium coli(choice B) is a ciliated organism that occasionally causes dysentery resembling that caused by Entamoeba histolytica.

Entamoeba histolytica(choice C) causes amoebic dysentery and liver abscesses.

Giardia lamblia(choice D) is a flagellated organism that infects the small intestine, causing diarrhea.

A sexually active 18-year-old woman presents with a fever of 102 F for the past 24 hours and lower abdominal pain and anorexia for the past 5 days. On physical examination, there is generalized tenderness of the abdomen,

Page 41: Microbiology II

and the cervix is erythematous with motion tenderness. There is no rash nor any lesions on the external genitalia. A smear of the odorless cervical discharge contains sloughed epithelial cells and scant neutrophils. Which of the following would likely be found in the exudate?

A. A naked, icosahedral double-stranded circular DNA virus

B. Iodine-staining intraepithelial inclusion bodies

C. Intraneutrophilic gram-negative diplococci

D. Intranuclear "owl's eye" inclusion bodies

E. Lactose-fermenting gram-negative bacilli

F. Pear-shaped flagellated protozoa

G. Pleomorphic, gram-negative rods

H. Spirochetes on dark-field microscopy

Explanation:

The correct answer is B. The presentation is typical for pelvic inflammatory disease (PID). Chlamydia trachomata (serotypes D-K) is the most common bacterial cause of sexually transmitted disease (STD) in this country and is the most likely agent on the list to produce the symptoms described. It is an ATP-defective organism that must therefore live intracellularly in the human host and can be visualized inside epithelial cells with iodine, Giemsa, or fluorescent-antibody stains. The remainder of the answer choices refer to other agents that could be found in the female genital tract, either by sexual transmission or by contamination with fecal flora, but they are not the best choices.

A naked, icosahedral double-stranded circular DNA virus (choice A) refers to human papilloma virus, which is the most common cause of STDs in the U.S., but presents with anogenital warts.

Intraneutrophilic gram-negative diplococci (choice C) refers to Neisseria gonorrhoeae, which would be expected to present with dysuria and neutrophilic exudate.

Intranuclear "owl's eye" inclusion bodies (choice D) refers to cytomegalovirus, a common STD in the United States, but not a common agent of PID. Most cases in average adults are manifested by mononucleosis-like symptoms.

Lactose-fermenting gram-negative bacilli (choice E) would be consistent with Escherichia coli. Although this

Page 42: Microbiology II

organism is the most common cause of urinary tract infections in women in the United States, it would not be expected to cause PID.

Pear-shaped flagellated protozoa (choice F) refers to the protozoan parasite Trichomonas vaginalis, the only protozoan STD in the world. Infection would be characterized by a malodorous, cheesy exudate, and there would be more erythema of the external genitalia than of the cervix.

Pleomorphic, gram-negative rods (choice G) are consistent with Hemophilus ducreyi, which causes chancroid, and presents in a different manner.

Spirochetes on dark-field microscopy (choice H) refers to Treponema pallidum, the causative agent of syphilis, which would produce rash and/or chancre, depending on the stage of the infection.

Which of the following microorganisms is an obligate aerobe?

A. Bacteroides fragilis

B. Clostridium perfringens

C. Escherichia coli

D. Mycobacterium tuberculosis

E. Pseudomonas aeruginosa

Explanation:

The correct answer is D.Mycobacterium tuberculosis is an important obligate aerobe to remember for USMLE Step 1. (Other obligate aerobes include all other Mycobacteria, Bordetella pertussis, and Francisella tularensis.). This organism can obtain energy only from respiration and must use oxygen as the terminal electron acceptor. Primary infection occurs in the upper lobes of the lung.

Bacteroides fragilis(choice A) is an obligate anaerobic gram-negative bacillus that constitutes the primary organism found in the colon. It is the most common cause of anaerobic infections, including abdominal abscesses and peritonitis.

Clostridium perfringens(choice B), along with all the other Clostridium species, is an obligate anaerobic, spore-forming, gram-positive bacillus. C. perfringens is an important cause of infection following trauma and surgery and can cause gas gangrene (which can be life-threatening if not treated). C. perfringens can also cause food poisoning.

Page 43: Microbiology II

E. coli(choice C) and P. aeruginosa(choice E) are both facultative organisms, meaning that they can grow both in the presence or absence of oxygen. E. coli can ferment under anaerobic conditions or respire using oxygen as the terminal electron acceptor. P. aeruginosa cannot ferment. It respires only, using nitrate as the terminal acceptor under anaerobic conditions and oxygen as the terminal electron acceptor under aerobic conditions.

A 27-year-old woman presents to the emergency department complaining of 10-12 episodes of nonbloody diarrhea per day for the past 2 days, along with severe abdominal cramps, nausea, vomiting, and a low-grade fever. She states that she just returned from a vacation to Mexico. While in Mexico, she did not drink any of the local water and ate only cooked foods and a few fresh salads. If fecal leukocytes are present, the patient should most likely be empirically treated with

A. acyclovir

B. ciprofloxacin

C. mebendazole

D. quinine

E. tetracycline

Explanation:

The correct answer is B. When individuals travel from one country to another with marked differences in climate and sanitation standards, the risk for development of traveler's diarrhea is dramatically increased. This type of diarrhea is most likely to develop within 2-10 days after ingesting local water and/or eating fresh fruits and vegetables (such as a fresh salad) that could have been "washed off" with the local water. Traveler's diarrhea is often accompanied by the appearance of approximately 10 or more episodes of diarrhea per day, as well as severe abdominal cramps, nausea, vomiting, and a mild fever. Depending on the causative bacteria, blood and/or fecal leukocytes may be present. Most cases of traveler's diarrhea are caused by enterotoxigenic Escherichia coli, Shigella species, and Campylobacter jejuni. The most commonly used agents to treat traveler's diarrhea are the fluoroquinolones, such as ciprofloxacin, ofloxacin, and norfloxacin. Trimethoprim/sulfamethoxazole is most commonly used in treatment of children. None of the other agents are indicated for the treatment of traveler's diarrhea.

Acyclovir (choice A) is an antiviral agent indicated for the treatment of herpes virus infections.

Page 44: Microbiology II

Mebendazole (choice C) is a broad-spectrum anthelminthic indicated for the treatment of "worm" infections, such as Enterobius vermicularis, Trichuris trichiura, and Ascaris lumbricoides.

Quinine (choice D) is an antimalarial agent indicated for the treatment of malaria and severe leg cramps.

Tetracycline (choice E) is an antibacterial agent with a limited antibacterial spectrum; it is primarily used in the treatment of acne vulgaris and gonorrhea infections.

Which of the following types of viruses would be most likely to undergo an abrupt, major antigenic shift permitting reinfection of previously exposed individuals?

A. Coxsackie viruses

B. Hepadna viruses

C. Herpesviruses

D. Orthomyxoviruses

E. Paramyxoviruses

Explanation:

The correct answer is D. The phrase "antigenic shift" should tip you off to look for "influenza viruses" or "orthomyxoviruses" in the answers. The orthomyxoviruses include influenza viruses A, B, and C. These viruses are successful at reinfecting previously exposed individuals because their genome consists of about eight (the viral packaging process is a little vague and sometimes entraps more pieces) separate segments of RNA. Minor antigenic changes are frequent and are called antigenic drift. Additionally, major changes, called antigenic shift rather than antigenic drift, rarely occur, possibly as a result of double infection of cells by human and animal influenza viruses with resulting accidental exchange of whole RNA segments. Such dramatic antigenic shifts have occurred four times for influenza A since 1933. The other viruses listed in the answers do not undergo such dramatic shifts.

A young mother takes her baby to the pediatrician for the first time. The doctor notices the infant's teeth have yellow discolorations. The antibiotic this mother most likely took during pregnancy

Page 45: Microbiology II

A. inhibits aminoacyl-tRNA binding

B. inhibits peptidyl transferase

C. interferes with cell wall synthesis

D. is a large, cyclic, lactone-ring structure

Explanation:

The correct answer is A. This question relates to a USMLE favorite side effect—the teeth mottling that occurs when a child is exposed to tetracycline in utero. You should remember that tetracycline is contraindicated in pregnancy and early childhood. Tetracycline is a bacteriostatic drug that binds to the 30s subunit of ribosomes, preventing aminoacyl-tRNA from binding with complementary mRNA. This inhibits peptide bond synthesis. Resistance is plasmid mediated.

Inhibition of peptidyl transferase (choice B) occurs with chloramphenicol, a broad-spectrum bacteriostatic agent that binds to the 50s subunit of ribosomes. Resistance is plasmid mediated. It has high toxicity (GI disturbances, aplastic anemia, and gray baby syndrome), so it is used mainly in severe infections or as a topical agent.

Interference with cell wall synthesis (choice C) occurs with penicillins and cephalosporins, the beta-lactam antibiotics. Resistance to these drugs appears in organisms that have developed beta-lactamases (penicillinases), enzymes that destroy the beta-lactam ring of these medications. The wider spectrum ampicillin, amoxicillin, ticarcillin, and carbenicillin are particularly penicillinase susceptible.

Large, cyclic, lactone-ring structures (choice D) describe the macrolides: erythromycin, azithromycin, and clarithromycin. They inhibit bacterial protein synthesis by reacting with the 50s ribosomal subunit and preventing the release of the uncharged tRNA. Resistance is plasmid mediated. Common side effects include GI irritation, skin rashes, and eosinophilia. Erythromycin is a popular choice for patients with penicillin hypersensitivity. It is a cytochrome p450 inhibitor and therefore must be used with caution in patients taking other drugs.

A 7- year-old girl develops behavioral changes, and her performance in school begins to deteriorate. Several months later she develops a seizure disorder, ataxia, and focal neurologic symptoms. She is eventually

Page 46: Microbiology II

quadriparetic, spastic, and unresponsive. Death occurs within a year. This patient may have had which of the following viral diseases at 1 year of age?

A. Chickenpox

B. German measles

C. Measles

D. Mumps

E. Parvovirus B19

Explanation:

The correct answer is C. The child has subacute sclerosing panencephalitis (SSPE), which is fortunately a very rare, late complication of measles infection at an early age. The existence of this complication is part of the rationale to immunize children to measles at an early age. It is thought that very young children's immune and neurologic systems may permit the virus to become established in the brain. The exact mechanism of injury is poorly understood, but the brain shows encephalitis involving both gray and white matter. 40% of cases die within 1 year, and it is thought that the disease is probably always eventually fatal.

None of the other diseases progresses to SSPE.

A 32-year-old man from Yuma, Arizona presents to a hospital with complaints of a 4-day history of fever, myalgias, and cough. He owns an automobile repair shop and had recently cleaned his garage, which was infested by mice. On physical examination, he is tachypneic and must use accessory respiratory muscles to breathe. Shortly following admission, he is intubated and diagnosed with adult respiratory distress syndrome. What underlying infection should be considered?

A. California encephalitis

B. Cytomegalovirus

C. Hantavirus

D. Poliovirus

E. Rabies

Explanation:

Page 47: Microbiology II

The correct answer is C. Hantavirus pulmonary syndrome, first reported in 1993, consists of fever, myalgia, and rapid development of respiratory failure leading to death. The initial cases were mostly among the Navajo Indians; however, cases have been documented in many states and in Canada. It is most common in New Mexico, Arizona, Colorado, and Utah. The causative agent is a Hantavirus belonging to the family Bunyaviridae. The virus is now called Sin Nombre (meaning "without a name") virus. Infection is thought to be through inhalation of aerosolized secretions from the common deer mouse. The disease typically begins with a non-specific prodrome (fever, generalized myalgia, and gastrointestinal disturbances) followed 4 to 5 days later by respiratory symptoms (cough, dyspnea, and tachypnea). This progresses rapidly to an adult respiratory distress syndrome and, in many cases, death.

California encephalitis (choice A) is caused by an arbovirus transmitted by infected mosquitoes. Most cases are in children less than 10 years of age. Signs and symptoms include fever, headache, photophobia, nausea, and vomiting. Myalgias and arthralgias are typically present. Seizure activity is most frequently observed in infants and young children. Most cases are in the midwestern U.S.

Cytomegalovirus (choice B) is a herpesvirus producing a variety of infections. Primary CMV infection is usually asymptomatic in immunocompetent patients, but can cause a heterophile-negative mononucleosis syndrome. Perinatal infections can occur in utero, intrapartum, or postpartum and can produce congenital malformations. CMV can be transmitted by granulocytes in blood transfusions. In patients with AIDS or other immunosuppressive states, CMV can produce severe disease, including retinitis, pneumonia, encephalitis, adrenalitis, and gastrointestinal tract ulcerations.

Poliovirus (choice D) is a member of the Picornavirus family. It remains epidemic in parts of Asia and Africa even though the wild-type has been eliminated from the western hemisphere. The virus affects cranial nerve nuclei and anterior horn motor neurons of the spinal cord, producing a flaccid paralysis, which is usually asymmetric. Vaccine-related polio can occur with the live virus vaccine.

Rabies (choice E) is a Rhabdovirus that produces encephalitis and/or myelitis. It should be considered in persons who have recently traveled outside of the U.S. The most common sources of exposure are dogs, cats, skunks, foxes, raccoons (in Florida and Connecticut), wolves, and bats. The infection is acquired by the bite of a rabid animal or by inhalation. Rabies also has been reported to occur in patients after corneal transplantation. The virus spreads along peripheral nerves to the central nervous system. Symptoms include hydrophobia and copious salivation.

Page 48: Microbiology II

A 55-year-old man returns from a vacation to the beach. While on vacation, he and his family consumed several meals consisting primarily of seafood, including lobster and raw oysters. Although the other members of his family did not get sick, he developed a bullous rash over his lower extremities, hypotension, and confusion. His past medical history is significant for liver disease and alcoholism. The most likely cause of this man's signs and symptoms is

A. Aspergillus fumigatus

B. Campylobacter jejuni

C. Candida albicans

D. Streptococcus (Group A)

E. Vibrio vulnificus

Explanation:

The correct answer is E.Vibrio vulnificus septicemia is contracted by consuming raw oysters. Patients with chronic renal insufficiency, liver disease, hematopoietic disorders, and a past history of alcoholism are particularly at risk. His past medical history makes him more susceptible to septicemia than are his family members. Septicemia cause by Vibrio vulnificus generally begins with chills, fever, and hypotension, and skin lesions tend to occur 24-48 hours after the onset of the infection.

Aspergillus fumigatus(choice A) is an opportunistic mold responsible for infections of wounds and burns in immunocompetent hosts; in immunocompromised hosts, it can invade visceral organs such as the lungs.

Campylobacter jejuni(choice B) are curved, gram-negative rods that cause enterocolitis with diarrhea, and, less commonly, chronic gastritis.

Candida albicans(choice C) is a common opportunistic yeast that is found as normal flora of upper respiratory, gastrointestinal, and vaginal mucosae. It ordinarily causes thrush and vulvovaginitis; it may disseminate in the immunocompromised patient.

Group A streptococcus (Streptococcus pyogenes; choice D) is an important cause of bacterial pharyngitis (Strep throat).

A 24-year-old female has fever, malaise, and a dry, nonproductive cough. She also complains of headache,

Page 49: Microbiology II

muscle aches, and leg pain. Lab values are significant for elevated cold agglutinins. Which of the following microorganisms is responsible for her symptoms?

A. Haemophilus influenzae

B. Klebsiella pneumoniae

C. Legionella pneumophila

D. Mycoplasma pneumoniae

E. Streptococcus pneumoniae

Explanation:

The correct answer is D.Mycoplasma pneumoniae is a wall-less bacterium that causes interstitial pneumonia in young adults. Elevated cold agglutinins (a classic clue) are found in about half of the patients. The cold agglutinins are IgM antibodies. M. pneumoniae may be diagnosed by sputum or complement fixation.

Haemophilus(choice A) causes bronchopneumonia in babies and children, and may occur in debilitated adults.

Klebsiella(choice B) causes a bronchopneumonia with patchy infiltrates involving one or more lobes. Think red currant jelly sputum. It frequently occurs in debilitated patients, diabetics, and alcoholics. Note that these organisms are highly encapsulated and produce mucoid colonies on lab media.

Legionella(choice C) lives in contaminated water sources such as air conditioning systems.

Streptococcus pneumoniae(choice E) is the most common cause of pneumonia in the elderly population and in those with poor nutrition. It may lead to a pleural abscess and rusty-colored sputum; it is also a common cause of sepsis and meningitis in the elderly.

A Native American man is brought to a rural hospital in New Mexico. On arrival, he is unconscious with severe bronchopneumonia. Family members state that he suffered the sudden onset of chills, fever, and headache several days ago. One day later, the man complained of chest pain and difficulty breathing, and coughed up blood-tinged sputum. Chest x-ray reveals patchy infiltrates and segmental consolidation. Which of the following organisms is the most likely cause of this man's pneumonia?

Page 50: Microbiology II

A. Brucella abortus

B. Clostridium perfringens

C. Francisella tularensis

D. Listeria monocytogenes

E. Yersinia pestis

Explanation:

The correct answer is E. Any previously healthy person in the Southwestern United States who develops septic shock or severe pulmonary disease should be evaluated for plague. Plague is not an extinct disease, but is still encountered in sporadic cases in various places in the world, including Asia, Africa, parts of Europe, and the American Southwest. The causative organism is Yersinia pestis, which is endemic in many wild animal populations, and can be transmitted to humans either by direct contact or by arthropod bite. Human plague may take many forms, including pestis minor (mild lymphadenopathy); bubonic plague (prominent lymphadenopathy); pneumonic plague (as in this patient); and septicemic plague. The primary pneumonic form typically presents as described. Antibiotics are most effective if given within the first 24 hours, which can be problematic if medical staff do not suspect the disease. Since plague is rare in the United States, a high degree of clinical suspicion is required to make a rapid diagnosis and to institute timely treatment. If the diagnosis is missed, the mortality rate is quite high.

Brucella abortus(choice A) causes brucellosis, characterized by undulating fever, lymphadenopathy, and hepatosplenomegaly.

Clostridium perfringens(choice B) causes gas gangrene and gastroenteritis.

Francisella tularensis(choice C) causes tularemia, associated with a spectrum of manifestations from an influenza-like syndrome to adenopathy with ulceration at the site of inoculation.

Listeria monocytogenes(choice D) causes listeriosis. Infection during pregnancy may result in sepsis, abortion or premature delivery. Infection in the neonate may produce meningitis. In immunocompromised adults, either meningitis or sepsis may occur.

An elderly patient has pneumococcal pneumonia. She has no known drug allergies. Which of the following antibiotics would be most appropriate?

A. Chloramphenicol

Page 51: Microbiology II

B. Cefotaxime

C. Erythromycin

D. Penicillin

E. Vancomycin

Explanation:

The correct answer is D. Penicillin remains the first-line drug of choice for pneumococcal pneumonia, except in those patients with penicillin allergy and in the relatively few areas in which pneumococcal strains with high-level penicillin resistance are found. Alternative therapies include erythromycin and vancomycin.

Chloramphenicol (choice A) is not usually used for pneumococcal pneumonia.

The third generation cephalosporin cefotaxime (choice B) is not usually used for pneumococcal pneumonia.

Erythromycin (choice C) is a good alternative therapy for pneumococcal pneumonia, but is usually used only when a penicillin allergy is present.

Vancomycin (choice E) is not the first-line therapy, but it is a good alternative in patients allergic to penicillin or when high-level penicillin resistance (relatively uncommon) is present.

A 30-year-old veterinarian visits her obstetrician for a first-trimester prenatal check-up. She has no complaints. Routine physical exam is significant only for mild cervical lymphadenopathy. She is prescribed spiramycin but is noncompliant. Her baby is born with hydrocephalus and cerebral calcifications. Which of the following organisms is most likely responsible?

A. Isospora belli

B. Leishmania donovani

C. Plasmodium vivax

D. Toxoplasma gondii

E. Trypanosoma cruzi

Explanation:

Page 52: Microbiology II

The correct answer is D. Humans become infected with Toxoplasma gondii by ingesting cysts in contaminated food or through contact with cat feces. The veterinarian in question was therefore particularly at risk of infection. T. gondii is especially hazardous in pregnant women because the organism can be transmitted to the fetus through the placenta. (It is part of the ToRCHeS group of congenital infections–Toxoplasma, Rubella, CMV, Herpes/HIV, Syphilis). Since infected mothers are usually asymptomatic, cases often go unnoticed. Occasionally, patients present with cervical lymphadenopathy, as did the veterinarian, and require treatment to prevent complications in the fetus. Though newborns are also often asymptomatic, they are at risk for developing the classic triad of chorioretinitis (at birth or later in life), hydrocephalus, and cerebral calcifications. Note that T. gondii is also a common cause of CNS infections (e.g., encephalitis) in HIV-positive patients.

Isospora belli(choice A) is an intestinal protozoan that causes watery diarrhea, particularly in the immunocompromised. Fecal-oral transmission of oocysts allows invasion of small intestinal mucosa, destroying the brush border.

Leishmania donovani(choice B) causes kala-azar (visceral leishmaniasis), which is characterized by fever, weakness, weight loss, splenomegaly, and skin hyperpigmentation. It is prevalent in regions of the Mediterranean, Middle East, Russia, and China. The vector is the sandfly.

Plasmodium vivax(choice C) causes malaria and is transmitted by the female Anopheles mosquito, which introduces sporozoites into the blood. These differentiate into merozoites that destroy erythrocytes. Splenomegaly ensues. Other species of this organism also cause malaria: P. malariae and P. falciparum (which causes a more severe form of the disease). Note that sickle cell trait confers resistance to this disease.

Trypanosoma cruzi(choice E) causes Chagas' disease, characterized by facial edema and nodules, fever, lymphadenopathy, and hepatosplenomegaly. It affects cardiac muscle most severely and is a major cause of cardiac disease worldwide. The reduviid ("kissing") bug is the vector that infects humans through bites. It is most prevalent in Central and South America, with rare cases in the southern US.

A 30-year-old veterinarian on a cattle ranch presents with a 1-to-2-month history of malaise, chills, drenching malodorous sweats, fatigue, and weakness. He has anorexia and has lost 15 pounds. He has intermittent fevers that range up to 103 F (39.4 C). He complains of visual blurring. A physical examination reveals mild lymphadenopathy, petechiae, and a cardiac murmur consistent with aortic insufficiency. What is the most likely etiologic agent?

Page 53: Microbiology II

A. Bacillus anthracis

B. Brucella abortus

C. Coccidioides immitis

D. Erysipelothrix rhusiopathiae

E. Trichinella spiralis

Explanation:

The correct answer is B.Brucella abortus produces a chronic, granulomatous disease with caseating granulomas. Most cases occur in four states (Texas, California, Virginia, and Florida), and are associated with cattle, in which it produces spontaneous septic abortions. Most cases of brucellosis produce mild disease or fevers of unknown origin. However, Brucella spp. can infect the cardiovascular system and cause a localized infection. B. abortus is the most common species to cause endocarditis. The aortic valve is most commonly involved, followed by the mitral valve, and then both valves. Most cases of brucellosis are associated with occupational exposure, in persons such as veterinarians, ranchers, and those who handle carcasses.

Bacillus anthracis(choice A) is the causative agent for anthrax. It usually produces cutaneous disease (malignant pustule or eschar) at the site of inoculation in handlers of animal skins. It can also produce a severe hemorrhagic pneumonia (Woolsorter's disease) and septicemia. At-risk groups include those who handle animal carcasses or skins.

Coccidioides immitis(choice C) is a dimorphic fungal disease producing a granulomatous pulmonary syndrome that is more severe in dark-skinned individuals. Disseminated disease occurs most often in Filipinos, Mexicans, and Africans. The infective form is the arthrospore; the diagnostic form in tissue is the spherule containing endospores. The disease is endemic in the San Joaquin River Valley. At-risk groups include military personnel, agricultural workers, construction workers, oil field workers, archaeology students, participants in outdoor sports, and sightseers. Remote infections from fomites (cotton harvested in the Southwestern U.S.) have been reported.

Erysipelothrix rhusiopathiae(choice D) is a pleomorphic, gram-negative rod that causes a localized skin infection. It is an occupational disease of fishermen, fish handlers, butchers, meat-processing workers, poultry workers, farmers, veterinarians, abattoir workers, and housewives.

Trichinella spiralis(choice E) is a nematode infection caused by the ingestion of larvae found in undercooked

Page 54: Microbiology II

meat. Pork is the most common contaminated meat. However, outbreaks in the northern parts of the U.S. have been associated with eating undercooked infected bear meat. Symptoms include diarrhea, periorbital edema, myositis, fever, and eosinophilia.

Six days after receiving several flea bites in a rat-infested shed in Southeastern New Mexico, a homeless 24-year-old-man develops fever, chills, and a rash that spreads from his abdomen to cover his extremities. He is seen at the hospital emergency department, where blood is drawn for analysis. Eight days later, the public health department reports the presence of antibody to one of the rickettsial group antigens. Which of the following is the most likely diagnosis?

A. Endemic typhus

B. Epidemic typhus

C. Q fever

D. Rocky Mountain spotted fever

E. Scrub typhus

Explanation:

The correct answer is A. Endemic typhus is caused by Rickettsia typhi and is found worldwide. It is spread by the feces of the rat flea, and its reservoir is the rat.

Epidemic typhus (choice B) is cause by Rickettsia prowazekii. It is prominent during times of war and social upheaval and is spread by the feces of the body louse, Pediculus humanis. Humans are the chief reservoir, although it is also found in populations of flying squirrels along the Atlantic coast of the U.S.

Q fever (choice C) is caused by Coxiella burnetii of the family Rickettsiaceae. Unlike the other rickettsial illnesses, Coxiella burnetii is not usually transmitted to humans by the bite of an arthropod. Since the organism is resistant to dehydration, it can be acquired by inhaling dust contaminated by animals with asymptomatic infections, such as goats, sheep, and cattle.

Rocky Mountain spotted fever (RMSF) (choice D) is caused by Rickettsia rickettsii. It is transmitted by the bite of an infected tick of the genus Dermacentor. Since the tick passes the infection transovarially to its progeny, the tick is a reservoir. Rodents and other mammals are also reservoirs. Despite the name, most cases of RMSF

Page 55: Microbiology II

occur in the central states and southern Atlantic seaboard states. Oklahoma has the highest incidence of the disease.

Scrub typhus (choice E) is caused by Rickettsia tsutsugamushi. It occurs mainly in Southeast Asia, where it is transmitted by mites. Mites and rodents are the reservoirs for the organism.

A 54-year-old diabetic patient reports to his physician's office complaining of an unresolved skin lesion on his foot. The lesion began several weeks ago as a blister and has since become a painful, erosive, expanding sore. On examination, the affected site is now 5 cm in diameter, with a black necrotic center and raised red edges. Which of the following toxins has a mechanism of action most similar to the toxin responsible for tissue damage in this patient?

A. Anthrax toxin

B. Botulinum toxin

C. Cholera toxin

D. Clostridium perfringens alpha toxin

E. Diphtheria toxin

F. Escherichia coli labile toxin

G. Pertussis toxin

H. Shigatoxin

I. Streptococcal erythrogenic toxins

J. Tetanus toxin

K. Toxic shock syndrome toxin-1

Explanation:

The correct answer is E. This patient's wound is infected with Pseudomonas aeruginosa, and the characteristic lesion described is called ecthyma gangrenosum. The Pseudomonas alpha toxin, which is responsible for the tissue damage, inhibits protein synthesis by acting on EF-2 with a primary target cell in the liver. The diphtheria toxin has a similar action, although its target cells are heart and nerve.

Anthrax toxin (choice A) is an adenylate cyclase that causes fluid loss from cells.

Page 56: Microbiology II

Botulinum toxin (choice B) is a neurotoxin that decreases acetylcholine synthesis.

Cholera toxin (choice C) acts to increase adenylate cyclase activity by ribosylation of GTP-binding protein.

Clostridium perfringens alpha toxin (choice D) is a lecithinase.

Escherichia coli labile toxin (choice F) works in a fashion similar to the cholera toxin.

Pertussis toxin (choice G) causes fluid loss by ribosylating Gi.

Shigatoxin (choice H) decreases protein synthesis by inhibiting the 60S ribosomal subunit.

Streptococcal erythrogenic toxins (choice I) act similarly to the diphtheria toxin, but do so by increasing cytokine production.

Tetanus toxin (choice J) is a neurotoxin that inhibits the inhibitory neurotransmitters glycine and GABA.

TSST-1 (choice K) is a superantigen that acts by increasing cytokine production and decreasing liver clearance of endotoxin.

An 18-year-old, previously healthy female presents to the student health service with fever, vomiting, and diarrhea. On physical examination, she is hypotensive and has an erythematous, red, sunburn-like skin rash. She is currently menstruating, and has been using super absorbent tampons. Which of the following findings from a positive blood culture would confirm your suspected diagnosis?

A. Organisms are acid-fast

B. Organisms are coagulase positive

C. Organisms grow on EMB (eosin-methylene blue) agar

D. Organisms grow on Thayer-Martin media

E. Organisms have positive Quellung reaction

Explanation:

The correct answer is B. This is a multi-step microbiology question that requires you to diagnose the illness, identify the microorganism, and remember its key feature. The first part should be easy: everything about this vignette suggests toxic shock syndrome. The organism in question is therefore Staphylococcus aureus, which is coagulase positive. All of the other choices are classic features of other important pathogenic microorganisms:

Page 57: Microbiology II

Acid-fast organisms (choice A), refers to Mycobacteria. (In addition, Nocardia species are partially acid fast).

EMB agar (choice C) refers to a selective and differential medium used to isolate and identify enteric gram-negative bacteria. Gram-positive bacteria will not grow on EMB agar because the addition of eosin inhibits their growth. Nonlactose fermenters will have colorless colonies, while fermentation of this sugar will cause the colonies to appear pink or purple.

Thayer-Martin media (choice D) is a growth medium for pathogenic Neisseria species. It contains the antibiotic vancomycin, which kills gram-positive organisms such as Staphylococcus aureus.

The Quellung reaction (choice E) can be used to identify the capsule type of a microorganism. Encapsulated microorganisms, like the Pneumococci and Haemophilus, are mixed with specific antisera. If the antiserum is directed against the microorganism's capsule type, the capsule will be opsonized, absorb water, and become visible under a light microscope.

A 24-year-old woman presents with a 3-day history of fever, chills, chest pain, and cough productive of rust-colored sputum. Past medical history includes a splenectomy 1 year ago. A chest x-ray film indicates consolidation of the right lower lobe. Blood cultures are positive for α-hemolytic gram-positive diplococci. Immunity to the causative organism is based on

A. alternative complement pathway activation

B. antibody to an α-helical coiled fimbria

C. IgA antibodies to C carbohydrate

D. IgG antibodies to C carbohydrate

E. IgG antibodies to a surface acidic polysaccharide

Explanation:

The correct answer is E. The patient in this question has pneumococcal pneumonia, which must be considered in any patient with chills, fever, chest pain, and cough productive of purulent, rust-colored sputum. Streptococcus pneumoniae is an α-hemolytic, gram-positive coccus that grows in chains. It can be easily distinguished from other α-hemolytic streptococci because it is exquisitely sensitive to bile and bilelike compounds, such as optochin. It is the most common cause of community-acquired pneumonia and the most common cause of community-acquired meningitis in adults older than 30. The only recognized virulence factor

Page 58: Microbiology II

of S. pneumoniae is its carbohydrate capsule (which contains acidic polysaccharides). Antibody to a specific capsule type is necessary to overcome infection. More than 80 capsule types have been recognized. The 23 types that most commonly cause disease are contained in a vaccine that is recommended for high-risk groups, including the elderly and those undergoing splenectomy. Increased susceptibility is also found in patients with Hodgkin disease, chronic lymphocytic leukemia, and myeloma. The vaccine should still be given to patients with these conditions, but it is less successful.

The alternative complement pathway (choice A) is important in clearing Neisseria infections. Individuals with deficiencies in C5 through C8 are at increased risk of disease from Neisseria.

The fimbria of Streptococcus pyogenes (Group A α-hemolytic streptococcus) is composed of an α-helically coiled M protein. Antibody against a specific M type (choice B) will prevent infection. However, raising antibodies to M proteins can lead to rheumatic fever, so Strep throat infections are routinely treated with penicillin to prevent an antibody response.

The C carbohydrate is an antigen of α-hemolytic streptococci used to divide them into different groups. Antibody against C carbohydrate (choices C and D) is not protective.

A 3-year-old boy presents with a 1-day history of loose stools, fever, abdominal cramping, headache, and myalgia. He has no blood in the stool. A careful history reveals that he has several pet turtles. Which of the following is most likely the causative agent of his diarrhea?

A. Chlamydia psittaci

B. Entamoeba histolytica

C. Salmonella spp.

D. Staphylococcus aureus

E. Yersinia enterocolitica

Explanation:

The correct answer is C. Salmonella spp., including S. enteritidis and S. typhimurium, produce a gastroenteritis or enterocolitis. Patients with decreased gastric acidity, sickle cell disease, defects in immunity, or children younger than 4 years have a more severe course of disease. Salmonella spp. are carried in nature by animal

Page 59: Microbiology II

reservoirs such as poultry, turtles, cattle, pigs, and sheep. The incubation period is 8-48 hours after ingestion of contaminated food or water.

Chlamydia psittaci(choice A) produces an interstitial pneumonitis accompanied by headache, backache, and a dry, hacking cough. A pale, macular rash is also found on the trunk (Horder's spots). Patients at risk include pet shop workers, pigeon handlers, and poultry workers.

Entamoeba histolytica(choice B) produces a diarrhea (frequently bloody or heme-positive), right lower quadrant crampy abdominal pain, and fever. Patients frequently have weight loss and anorexia. There is usually a history of travel outside the U.S. Most cases are chronic. Complications include liver abscesses.

Staphylococcus aureus(choice D) produces a self-limited gastroenteritis due to the production of preformed, heat-stable enterotoxins. The incubation period is 16 hours. The toxins enhance intestinal peristalsis and induce vomiting by a direct effect on the CNS.

Yersinia enterocolitica(choice E) usually produces a chronic enteritis in children. These patients have diarrhea, failure to thrive, hypoalbuminemia, and hypokalemia. Other findings include acute right lower quadrant abdominal pain, tenderness, nausea, and vomiting. The infection mimics appendicitis or Crohn's disease.

Bilateral tonsillectomy is performed on an otherwise healthy 11-year-old female with recurrent upper respiratory tract infections. On sectioning the tonsils, numerous small, yellow granules are noted. A granule crushed between two slides has a dense, gram-positive center and numerous branching filaments at the periphery. The granules are most likely composed of which of the following organisms?

A. Actinomyces israelii

B. Aspergillus fumigatus

C. Blastomyces dermatitidis

D. Candida albicans

E. Corynebacteria diphtheriae

Explanation:

The correct answer is A. Actinomyces are normal inhabitants of the gastrointestinal tract that grow under anaerobic and microaerophilic conditions. Although they are gram-positive rods, they grow as branching

Page 60: Microbiology II

filaments and have been confused with fungi. The yellow colonies (sulfur granules) are found in low-oxygen niches like the tonsils and in actinomycotic abscesses.

Aspergillus fumigatus(choice B)may be present in the respiratory tract as an opportunistic pathogen; however, fungus balls are generally seen only in pre-existing cavities (e.g., bronchiectasis, TB), not in the tonsils.

Blastomyces dermatitidis(choice C) is a respiratory pathogen that is seen as thick-walled yeasts within granulomas.

Candida albicans(choice D), also a normal inhabitant of the oral cavity, would present as whitish plaques and would appear microscopically as budding yeasts.

Diphtheria, caused by Corynebacteria diphtheriae(choice E), is a gram-positive rod. The disease presents with a gray-white membrane in the oropharynx, and large colonies would not be appreciated.

A patient with a cavitary lung lesion coughs up sputum that contains thin, acid-fast positive rods. Which of the following features would most likely be associated with these bacteria?

A. Nutritional requirement for factors V and X

B. Streptokinase

C. Toxic shock syndrome toxin

D. Visible under dark field illumination

E. Waxy envelope

Explanation:

The correct answer is E. Mycobacteria, such as the causative organism of this patient's tuberculosis, are "acid fast" because they have an envelope that contains large amounts of lipids and even true waxes (unlike envelopes of other types of bacteria) that prevents the acid-fast stain (carbolfuchsin) from leaking out.

The other characteristics listed in the answers are commonly tested features of specific bacteria:

Nutritional requirement for factors V and X (choice A) is a feature of Haemophilus influenzae.

Streptokinase (choice B) is a feature of Streptococci.

Toxic shock syndrome toxin (choice C) is a feature of Staphylococcus aureus.

Page 61: Microbiology II

Visibility under dark field illumination (choice D) is a feature of the syphilis organism Treponema pallidum.

An 8-year-old boy is brought to the emergency room with a 3-day history of fever to 102 degrees F and abdominal pain. He also complains of pain in his right knee and right elbow. He was seen four weeks ago because of a sore throat and a rash. A throat culture performed at that time grew gram-positive cocci in chains. Amoxicillin was prescribed, but the boy's mother did not fill the prescription. On physical exam his temp is 101.7F, HR 96, and BP 100/60. Cardiac exam reveals a pansystolic blowing murmur heard best at the apex. His right elbow is tender on extension and flexion with mild swelling. Laboratory tests reveal a positive C-reactive protein, an ESR of 40 and a WBC of 22,000 with a left shift. EKG shows a prolonged PR interval. Which of the following tests would be positive for the microorganism responsible for this patient's illness?

A. Catalase test

B. Coagulase test

C. Sensitivity to bacitracin

D. Sensitivity to novobiocin

E. Sensitivity to optochin

Explanation:

The correct answer is C. This case is classic for Rheumatic fever, including two of the major Jones criteria (carditis, polyarthritis) and several minor criteria (fever, arthralgia, elevated ESR, leukocytosis, C-reactive protein, prolonged PR interval). Rheumatic fever is a sequela of untreated infection with Group A Streptococcus (S. pyogenes). S. pyogenes is differentiated from the other beta-hemolytic Strep by its sensitivity to the antibiotic bacitracin.

The catalase test (choice A) is used to differentiate Staphylococci from Streptococci. Staphylococci are catalase positive, Streptococci are catalase negative. So, S. pyogenes would be catalase negative.

The coagulase test (choice B) is used to differentiate Staph aureus from the other Staph spp. Staph aureus is coagulase positive; the others are negative.

Sensitivity to novobiocin (choice D) is used to differentiate Staph saprophyticus (resistant) from Staph epidermidis (sensitive).

Page 62: Microbiology II

Sensitivity to optochin (choice E) is used to differentiate Strep pneumoniae (sensitive) from viridans Strep (resistant).

An immigrant from the Far East develops malaise, fever, and rigors, followed by upper right quadrant abdominal pain, vomiting, jaundice, and itching. His urine is dark and his feces are pale. Infestation with which of the following parasites is most strongly suggested by this patient's presentation?

A. Clonorchis sinensis

B. Enterobius vermicularis

C. Plasmodium ovale

D. Taenia solium

E. Trypanosoma cruzi

Explanation:

The correct answer is A. The patient is suffering from suppurative (bacterial) cholangitis, which can occur as a complication of infestation by the roundworm, Ascaris lumbricoides and by the liver flukes, Clonorchis sinensis and Fasciola hepatica. Biliary tract obstruction produces jaundice and extreme itching, with dark urine and pale feces. Therapy typically includes emergency endoscopic sphincterectomy to improve biliary drainage, antibiotics, and anthelminthic agents.

Enterobius vermicularis(choice B) causes pinworm infections.

Plasmodium ovale (choice C) causes malaria.

Taenia solium (choice D) is the pork tapeworm. Adult tapeworms cause taeniasis, while the larvae are responsible for cysticercosis.

Trypanosoma cruzi(choice E) causes Chagas' disease.

A 38-year-old woman vacationing in Connecticut is bitten by a tick. She does not seek medical treatment and eventually develops chronic arthritis of the knee and hip joints and paralysis of the left facial muscles. A physical examination during the early stages of the disorder would most likely have revealed

A. aphthous ulcers in the mouth

Page 63: Microbiology II

B. erythema chronicum migrans

C. flaccid paralysis of limb flexors

D. purpuric lesions in a bathing trunk distribution

E. spastic paralysis of limb extensors

Explanation:

The correct answer is B. Lyme disease should be suspected in a patient who is bitten by a tick in the northeastern U.S. Lyme disease was named after a township in eastern Connecticut where the disease was endemic. The disease is spread via a tick vector of the genus Ixodes, which transmits a spirochete that causes a systemic illness. Erythema chronicum migrans is usually the first sign of the illness. This is a large red patch on the buttocks or chest that slowly expands as the center blanches. Generally, patients also have constitutional symptoms, such as fever and chills, during this phase. Stiff neck may develop, along with other signs of meningeal irritation, because of an aseptic meningitis. Other neurologic complications of Lyme disease include Bell's palsy due to involvement of branches of the facial nerve. Arthritis is a prominent feature in about half the patients with Lyme disease. It tends to appear several months after the infection but may persist for several years. The course of the chronic arthritis shows exacerbations and remissions; the most commonly affected joints are the knees and hips. Cardiac abnormalities in Lyme disease include pericarditis and heart block.

Skin manifestations do not include aphthous ulcers (choice A).

Flaccid or spastic paralysis of limbs (choices C and E) does not accompany Lyme disease; neurologic involvement is generally limited to cranial nerves and meningitis.

Purpura (choice D) is associated with vasculitis and does not occur in Lyme disease.

Global eradication of Lyme disease is unlikely because

A. Borrelia burgdorferi can be maintained in nature indefinitely by a tick vector

B. Borrelia burgdorferi is resistant to antibiotics and disinfectants

C. Borrelia burgdorferi is resistant to environmental stresses

D. human disease may reactivate after the primary infection (Brill-Zinsser disease)

E. humans are the primary reservoir for Borrelia burgdorferi

Explanation:

Page 64: Microbiology II

The correct answer is A.Borrelia burgdorferi can be maintained in nature indefinitely by a tick vector. This organism is the tick-transmitted spirochete that causes Lyme disease. The tick, Ixodes dammini, can infect both the white-footed mouse and large mammals such as deer during its life cycle, making them reservoirs. The tick itself is a reservoir, however, since it acquires the disease through transovarial passage of the organism. Together, these factors make Lyme disease an endemic infection with little hope for eradication.

Borrelia burgdorferi is not resistant to antibiotics and disinfectants (compare with choice B). The spirochete can be successfully treated with penicillins, tetracycline, and ceftriaxone.

Borrelia burgdorferi is a delicate spirochete and is not resistant to environmental stresses (compare with choice C).

Brill-Zinsser disease (choice D) is the reactivation of epidemic typhus infection caused by Rickettsia prowazekii. It can occur many years after an infection that was not treated with antibiotics.

Humans are incidental hosts, rather than the primary reservoir, for Borrelia burgdorferi(choice E). The primary reservoirs are ticks, mice, and large mammals.

A 54-year-old farmer in rural Pennsylvania presents to his physician with chronic cough. Chest x-ray demonstrates a mass lesion with hilar lymphadenopathy. Biopsy of the mass demonstrates multiple, tiny yeast forms within macrophages. Which of the following is the most likely diagnosis?

A. Blastomycosis

B. Coccidioidomycosis

C. Histoplasmosis

D. Paracoccidioidomycosis

E. Sporotrichosis

Explanation:

The correct answer is C. Histoplasmosis is caused by Histoplasma capsulatum, a dimorphic fungus that grows as a mold in the wild, but as a tiny yeast inside macrophages in humans. The disease is typically asymptomatic or mild enough to go undetected, but when symptomatic, it presents with cough, fever, and malaise.

Page 65: Microbiology II

Blastomycosis (choice A) is characterized by a larger, round-budding yeast form, seen free in the tissues.

Coccidioidomycosis (choice B) is seen mainly in the desert parts of the Southwest U. S. In the lungs, spherules containing endospores are seen.

Paracoccidioidomycosis is endemic in Latin America, especially in Brazil. The infected cells show a typical "pilot's wheel" appearance due to multiple yeasts sprouting out of a single parent cell (choice D).

Sporotrichosis (choice E) most often produces a localized cutaneous infection, following inoculation occurring in association with minor skin trauma during gardening.

Two weeks after birth, a neonate develops sepsis, skin vesicles, and conjunctivitis. Over the next several days, the baby's condition deteriorates with development of seizures, cranial nerve palsies, and lethargy. The baby dies approximately one week after onset of symptoms. Which of the following infectious agents would most likely cause this clinical presentation?

A. Cytomegalovirus

B. Herpes simplex

C. Rubella

D. Syphilis

E. Toxoplasmosis

Explanation:

The correct answer is B. All of the agents listed, including choices A, C, D and E, can cause devastating congenital infections with high mortality and often with major organ malformation (the TORCH agents: Toxoplasma, other, rubella, cytomegalovirus, herpes simplex). However, it is herpes simplex type II, typically acquired during delivery, that causes the devastating neonatal encephalitis described in the question stem. The mortality rate for neonatal herpes is about 65%, and only 10% of the babies escape without neurologic sequelae. A point worth remembering is that adult herpes encephalitis (in non-immunosuppressed individuals) is usually due to herpes simplex I, while neonatal and congenital herpes are usually due to herpes simplex II.

Page 66: Microbiology II

A one-week-old baby develops nuchal rigidity and fever. A lumbar puncture is performed and the cerebrospinal fluid demonstrates large numbers of neutrophils. Which of the following is the most likely causative agent?

A. Coxsackievirus

B. Escherichia coli

C. Herpes virus

D. Mycobacterium tuberculosis

E. Neisseria meningitidis

Explanation:

The correct answer is B. The nuchal rigidity (stiff neck) suggest meningitis. Numerous neutrophils in the cerebrospinal fluid suggests a bacterial pathogen. The best answer of those listed is Escherichia coli, which is normally a gut organism, but can infect neonates who acquire the organism during passage through the birth canal.

Coxsackievirus (choice A) is a cause of acute lymphocytic meningitis.

Herpes virus (choice C) is a cause of lymphocytic meningitis.

Mycobacterium tuberculosis(choice D) is a cause of chronic meningitis.

Neisseria meningitidis(choice E) causes bacterial meningitis, usually in the second or third decade of life.

A patient presents to a physician because of pain during defecation accompanied by blood in the stool. Physical examination demonstrates a large perianal mass. Pathologic examination of the rectal mass following resection demonstrates a condyloma in which transformation to frank carcinoma has occurred. Which of the following viruses would most likely be associated with these lesions?

A. Epstein-Barr virus (EBV)

B. Hepatitis B virus (HBV)

C. Human herpesvirus type 8 (HHV8)

D. Human papilloma virus (HPV)

E. Human T-cell leukemia virus (HTLV-1)

Page 67: Microbiology II

Explanation:

The correct answer is D. The tumor is anal carcinoma arising in a condyloma. Both condyloma and anal carcinoma are related to human papilloma virus (HPV), which is also associated with cervical and penile condylomas and carcinomas.

EBV (choice A) is associated with Burkitt's lymphoma and nasopharyngeal carcinoma.

HBV (choice B) is associated with hepatocellular carcinoma.

HHV8 (choice C) is associated with Kaposi's sarcoma.

HTLV-1 (choice E) is associated with adult T-cell leukemia.

A 23-year-old woman with a history of sickle cell disease presents with fever and severe bone pain localized to her left tibia. X-ray reveals a lytic lesion and blood cultures reveal infection. A bone culture grows gram-negative rods. Which of the following best describes the infecting organism?

A. It is a facultative intracellular parasite

B. It is a nonmotile facultative anaerobe

C. It is comma-shaped and sensitive to acidic pH

D. It is motile and does not ferment lactose

E. It is motile and oxidase positive

Explanation:

The correct answer is D. The presence of sickle cell disease in a question stem is usually a significant clue. This question tests if you know that patients with sickle cell anemia are more susceptible to osteomyelitis caused by Salmonella. (The patient's fever, bone pain, and x-ray results indicate osteomyelitis). But note that Staphylococcus aureus (gram-positive coccus) is the most common cause of osteomyelitis in both sicklers and nonsicklers. If it had not been ruled out on bone culture, you should have looked for it in the answer choices. Notice that you were required to know more than just the organism's name; you needed to know its distinguishing features. Choice D describes Salmonella (a gram-negative rod) accurately. Salmonella exists in over 1800 serotypes and is known to contaminate poultry.

A facultative intracellular parasite (choice A) is Legionella, a catalase-positive gram-negative rod. It

Page 68: Microbiology II

contaminates air-conditioning cooling towers and causes Legionnaire's disease (a type of pneumonia).

A nonmotile, facultative anaerobe (choice B) is Shigella, a gram-negative rod that does not produce H2S. All Shigella contain an endotoxic lipopolysaccharide. The organism causes bacillary dysentery, with abdominal cramps, fever, and mucoid, bloody diarrhea.

A comma-shaped organism that is sensitive to acidic pH (choice C) is Vibrio cholerae, a gram-negative rod that causes severe enterotoxin-induced diarrhea, with "rice-water" stools and dehydration. The toxin acts by stimulating adenylyl cyclase to overproduce cAMP in the brush border of the small intestine.

A motile and oxidase positive organism (choice E) is Pseudomonas, a gram-negative rod with pili that sometimes produces a polysaccharide slime layer. P. aeruginosa is the prototype and commonly colonizes the lungs of patients with cystic fibrosis. It is associated with blue-green pus.

A 42-year-old Hispanic man is brought to the emergency room by ambulance after suffering a grand mal seizure at home. There is no history of recent illness, fever, headache, seizures, or head trauma. He drinks alcohol occasionally and denies any other drug use. The patient has been a resident of the United States for 15 years, but occasionally travels to his previous home in Honduras. Neurologic exam shows the patient to be alert and oriented. No focal abnormalities are noted. A CT scan of the head reveals multiple punctate calcifications, and two enhancing cystic lesions with surrounding edema. What is the most likely diagnosis?

A. Amebiasis

B. Cytomegalovirus infection

C. Echinococcosis

D. Neurocysticercosis

E. Toxoplasmosis

Explanation:

The correct answer is D. Cysticercosis is a parasitic infection caused by the larval cysts of the tapeworm Taenia solium. Patients acquire the infection by ingesting the eggs, which reach a larval stage in various tissues. When the central nervous system (CNS) is involved, the condition is known as neurocysticercosis. It is the most common parasitic infection of the CNS. Infection with this organism is most frequently encountered in individuals

Page 69: Microbiology II

from Mexico, South Central America, the Philippines, and Southeast Asia. In the CNS, the cysts act as space-occupying lesions and can cause hydrocephalus and/or seizures. Seizures are the most common initial presentation of patients with neurocysticercosis and may be focal or generalized. Signs of increased intracranial pressure such as headache, nausea, vomiting, or visual changes may also be present.

Amebiasis (choice A) is caused by Entamoeba histolytica. Patients typically present with diarrhea (often bloody), right lower quadrant abdominal pain, and fever. Amebic abscesses in the liver are a complication due to invasion of the portal venous system by the amoeba.

Cytomegalovirus (choice B) produces neonatal infections and infections in immunocompromised patients such as AIDS patients. CD4 counts are usually <100 cells/mm3. The most common clinical presentation is chorioretinitis producing floaters, visual field deficits, and painless loss of vision. CMV also produces encephalitis and may produce calcifying lesions in the CNS.

Echinococcosis (choice C) is a parasitic nematode infection caused by either Echinococcus granulosis or E. multilocularis. The disease is hydatid cyst disease. The patient ingests the eggs from dogs and becomes an intermediate host. The cysts are classically in the liver, are calcified, and the patient shows eosinophilia.

Toxoplasmosis (choice E) is a protozoan infection acquired by eating undercooked meat or by exposure to cat feces. Primary toxoplasmosis is usually asymptomatic. In patients with normal immunity, the organism can cause a heterophile-negative, mononucleosis-like syndrome. In patients with AIDS, it causes ring-enhanced focal brain lesions and pneumonia.